You are on page 1of 77

[1]

CONTENTS
WORK AND ENERGY
1. Work..................................................................................................................................................... [2]
1.1 Work done by constant force.......................................................................................................... [2]
1.2 Work done by variable force........................................................................................................... [2]
2. Various work performing agents....................................................................................................... [5]
2.1 Work done by gravity...................................................................................................................... [5]
2.2 Work done by tension of a string ................................................................................................... [5]
2.3 Work done by friction .................................................................................................................... [5]
2.4 Work done by spring ..................................................................................................................... [6]
2.5 Work done by internal forces......................................................................................................... [7]
3. Power .................................................................................................................................................. [8]
IN CHAPTER EXERCISE 1 ......................................................................................................... [10]
4. Energy ................................................................................................................................................ [16]
5. Work Energy Theorem for a particle ............................................................................................. [17]
6. Work energy theorem for a system of particles ............................................................................. [19]
7. Types of forces ................................................................................................................................... [23]
7.1 Conservative Forces ...................................................................................................................... [23]
7.2 Non-conservative forces ............................................................................................................... [23]
8. Potential energy ................................................................................................................................. [24]
8.1 Potential energy of System ........................................................................................................... [24]
8.2 Potential energy of Particle ........................................................................................................... [25]
9. Conservation of Energy .................................................................................................................... [27]
9.1 Gravitational Potential Energy ...................................................................................................... [27]
9.2 Spring Potential energy ................................................................................................................. [29]
IN CHAPTER EXERCISE-2 ........................................................................................................... [30]
10. Application of Energy conservation in circular motion ............................................................... . [36]
10.1 Circular motions with normal force ............................................................................................ [36]
10.2 Circular motions with Tension force .......................................................................................... [36]
11. Relation between Potential energy and Force ................................................................................ [40]
11.1 Potential energy if force field is given ........................................................................................ [40]
11.2 Force if potential energy is given ................................................................................................ [42]
11.3 Force from graph of potential energy .......................................................................................... [42]
12. Nature of Equilibrium ..................................................................................................................... . [43]
12.1 Stable Equilibrium ..................................................................................................................... . [43]
12.2Unstable equilibrium ................................................................................................................... . [43]
12.3Neutral equilibrium...................................................................................................................... .. [43]
IN CHAPTER EXERCISE-3 ....................................................................................................... . [45]
Exercise-1........................................................................................................................................... . [52]
Type-1.................................................................................................................................................. [52]
Type-2.................................................................................................................................................. [57]
Exercise-2 [JEE Main/AIEEE Asked] .......................................................................................... . [64]
IIT - JEE/ JEE Advance Asked………………………………………………………………….. . [69]
Exercise-3.......................................................................................................................................... . [73]

Raman Niwas, Near Aakashvani, Mahmoorganj, Varanasi. Ph.(0542)–2363455,website www.catjee.in


[2]

Work and Energy


1. Work

The work dW performed by a force F acting on an object when its point of application on the object

undergoes a displacement ds is defined as:
 
dW  F  ds
Work is a scalar quantity having SI unit of Newton-meter also called Joule. The work done by a given
force on a body depends only on the force, the displacement, and the angle between them. It does not
depend on the velocity or the acceleration of the body, or on the presence of other forces. Since
displacement of a point depends on chosen frame of reference, work done by a force also depends on
chosen frame of reference. Depending on the angle  between the force and corresponding
displacement vector, the work done by a force on an object may be positive, negative or zero. Thus,

For 0    , work done is positive.
2

For   , work done is zero.
2

For     , work done is negative.
2
For example,
(i) When a person lifts a body from the ground, the work done by the lifting force is positive but
the work done by the gravitational force is negative.
(ii) When a body slides on a fixed rough surface, work done by the pulling force is positive while
work done by the force of friction is negative. The work done by normal force is zero.
1.1 Work done by constant force: If work is being performed by a force which is constant in
magnitude as well as in direction, then the total work by the force can be determined as:
 
dW  F  ds
W f  
 dW
0
  F
i
 ds [i and f represent initial and final conditions]
 f 

 W  F    ds 
 i 
 
 W  F  s
 
 W  F s cos

The above equation can be considered as either product of magnitude of force and component
of displacement in the direction of force or product of magnitude of displacement and
component of force in the direction of force.
1.2 Work done by variable force: To find work performed by a variable force we consider an

infinitesimal displacement ds and determine the work dW during this displacement. Large
number of such displacements is then considered and individual works are then summed up to
get total work performed. This process of summing up large number of small quantities
involves integration.

Raman Niwas, Near Aakashvani, Mahmoorganj, Varanasi. Ph.(0542)–2363455,website www.catjee.in


[3]
Example 1: A box is moved a horizontal path by applying force F  60N at an angle   30 to the
horizontal. What is the work done during the displacement of the box over a distance of 0.5m?
Solution: Since the given force is constant,
W  F s cos
Here F  60 N; s  0.5m;   30
 W   60 0.5 cos30  26J

Now Solve: A box is moved on a vertical path by applying force F  60N at an angle   30 to the
horizontal. What is the work done during the displacement of the box over a distance of 0.5m?
Ans: 15J
Example 2: A block of mass m  3000kg is lifted by a string with acceleration a  2m/s2 . Find work done
by string on block during 2s from the beginning of motion.
Solution: The height h to which the body is lifted during the first t second is
1
h  at 2  4m
2
Using free body diagram:
T

mg

The tension in the spring is given by


T  mg  ma

 T  30000N   3000kg   2m/s2 

 T  36000N
Hence, work done is given by
W   36000N  4m   144kJ

Now Solve: A block of mass m  1000kg is lowered down by a string with acceleration a  3m/s2 . Find
work done by string on block during 2s from the beginning of motion. Also solve the problem
with respect to string.
Ans: 4200 J, 0

Example 3: A force F acts on a particle with position x given by

F   xi
Here  represents a positive constant. As the particle moves from position x1 to x2, find work
performed by this force.
Solution: Considering an elemental displacement dxi when the particle has reached position x, the work
done dW during this displacement is given by:

  
dW   xi  dxi   x  dx

Raman Niwas, Near Aakashvani, Mahmoorganj, Varanasi. Ph.(0542)–2363455,website www.catjee.in


[4]
Performing integration both sides with proper limits to get the total work W by this force,

 x  dx
W x2
0
dW  
x1


 W
2
x 2
2  x12 

Now Solve: Find out the work done by spring of spring constant k when its extension is changed from x1 to
x 2.

W   k  x22  x12 
1
Ans:
2
Example 4: A body placed on a smooth horizontal surface experiences a horizontal force varying with
displacement of block as shown in graph:
F

30N
20N

10N
x
1m 2m 3m 4m 5m 6m 7m

Find the total work performed by given force.


Solution: Using the result for the work performed by a variable force, we can conclude that the graph of
force with displacement can be used to find total work by finding total area under the curve of
the graph.
Thus total work is given by:
1 1 1
W   3  30   1 50  20  2   2  20  130J
2 2 2
Now Solve: Solved the previous problem for the graph given below:
F

30N
20N

10N

x
1m 2m 3m 4m 5m 6m 7m 8m 9m

Find the total work performed by given force.


Ans: 45 J

Raman Niwas, Near Aakashvani, Mahmoorganj, Varanasi. Ph.(0542)–2363455,website www.catjee.in


[5]
2. Various work performing agents
For practical purpose, work done is often determined by following few standard force applying agents:
2.1 Work done by gravity: The work done by gravity can be determined by using the result for
work done by a constant force. Thus, for an object of mass m which descends a vertical height
h, then during this process work done by gravity is
WGravity  mgh

Such work by gravity will be negative if object moves upwards. For this result, the acceleration
due to gravity has been assumed to remain constant.
2.2 Work done by tension of a string: Considering string to be mass less and inextensible, the
sum of works performed by string on the two particles attached to its two ends, is always zero.
This is because the string always pulls both ends with same force, but one end gets pulled and
other end moves away (as length of string has to remain constant). However, work performed
by one end of the string on the particle attached to it may be positive, negative or zero.
2.3 Work done by friction: If frictional force is applied by surface S on a block B, then work done
by this frictional force relative to surface S is either zero (if static frictional force acts) or
negative (if kinetic frictional force acts). However, this work may be positive, negative or zero
relative to any other considered frame of reference.

If object A applies frictional force f on object B, during which there occurs displacements of
 
 sA and  s B for the two objects relative to an arbitrary frame of reference S, then total work
performed by friction relative to this frame S is:
 on A   on B 
Total
Wfriction  Wfriction  Wfriction
   
 Total
Wfriction     
  f   sA  f   s B 
  
 Total
Wfriction  
 f   sA   s B
 
 Total
Wfriction  
 f   s B, A 

Here,  s B, A represents the displacement of B relative to A. Now due to nature of frictional
  
force, we conclude that f acts on B opposite to  s B, A (or  s B, A  0 when no relative slipping
occurs ). Hence,
 Total 
WFriction 0
So, we can say that if friction acts between objects A and B, the sum of works performed by
friction on A and B is either zero (if no relative slipping occurs) or negative (when slipping
occurs). This is to be noted that frictional force may perform positive, negative or zero force on
an object.
Example 5: A boy pulls a 5 kg block 20m along a horizontal surface at a constant speed with a force
directed 45° above the horizontal as shown:
F



If the coefficient of kinetic friction is 0.2, find the work done by


(i) Gravitational force on the block.
(ii) Frictional force on the block.
(iii) The boy on the block.

Raman Niwas, Near Aakashvani, Mahmoorganj, Varanasi. Ph.(0542)–2363455,website www.catjee.in


[6]
Solution: The forces acting on the block are shown in figure:
F sin  F
N


F cos 
N

mg

As the block moves with uniform velocity, we have


N  F sin 45  mg
And F cos 45   N
From above equations we get
 mg
F  11.55 N
cos 45   sin 45
And the frictional force f is given by,
f   N  F cos 45  8.17N
(i) As the block gets displaced horizontally while gravitational force acts vertically
downwards, the work done by gravitational force will be zero.
(ii) The block is pulled through a horizontal distance 20.0 m. Thus the work done by
friction is

W  8.17N  2.0m cos180o  163.3J

(iii) The work done by force F is given by,


W  11.55N  2.0m cos45o  163.3J
Now Solve: A boy pulls a 5 kg block 20m on an inclined plain at a constant speed with a force directed 45°
as shown in figure:
F



If the coefficient of kinetic friction is 0.2, find the work done by


(i) Gravitational force on the block.
(ii) Frictional force on the block.
(iii) The boy on the block.
Ans: (i) – 600 J, (ii) – 160 J, (iii) + 760 J
2.4 Work done by spring: Spring is an object which applies force on objects depending on
relative separation between its ends. If spring is elongated from natural length, it pulls its ends
towards itself. If the spring is compressed from its natural length, it pushes away its two ends.
Considering such a spring,

Raman Niwas, Near Aakashvani, Mahmoorganj, Varanasi. Ph.(0542)–2363455,website www.catjee.in


[7]
Equilibrium

k N

kx F
x
k
F
mg

dx

As the block moves slowly due to application of applied force F the length of spring changes.
As the length of spring changes magnitude of F increases to balance the larger force applied by
the spring. Thus, we need to perform integration to find work done by spring force or the
applied force F. Consider an elemental displacement dx after an elongation of x. work
performed by spring during this displacement is:

dWspring   kx  dx   cos180o   kxdx


Wspring xf
0
dWSpring   kxdx
xi

Wspring   k  x 2f  xi2 
1
2
If spring was initially un-stretched and undergoes a total elongation of xo,
1
Wspring   k xo2
2
Similarly,

WF  k  x 2f  xi2 
1
2
2.5 Work done by internal forces: Vector sum of all internal forces on a system is zero. But work
performed by internal forces on a system may not be zero. Consider a system of objects A and
 
B such that object A applies a force F AB on object B, and there occurs a displacement of S A and

S B respectively for the blocks A and B. The net work done by pair of these forces is:
   
Wnet  F AB  S   F  S 
A BA B
  
 Wnet  F AB  S  S  A B
[Using Newton‘s III law]
 
 Wnet  F AB  S  AB

Thus if there occurs a relative displacement between objects A and B, the pair of forces of
interaction between A and B performs total of non-zero work. This can be observed that such
total work performed by this pair of forces is independent of frame of reference.

Raman Niwas, Near Aakashvani, Mahmoorganj, Varanasi. Ph.(0542)–2363455,website www.catjee.in


[8]
3. Power
Average Power developed by a force in a time interval is defined as the ratio of total work done by
force in that time interval and the time interval. Instantaneous power is average power in an
infinitesimally small time interval or simply the rate at which the given force performs work.
Mathematically,
W
Paverage 
t
W dW
And Pinstantaneous  lim 
t 0 t dt
Power is a scalar physical quantity and has a SI unit Joule/second also called Watt. For instantaneous
power, following result may also be developed:
 
dW F  ds  
Pinst    F v
dt dt
 
Here v represents instantaneous velocity of the point of application of the force F .
Example 6: A body of mass m is thrown at an angle  to the horizontal with the initial velocity u. Find the
average power developed by gravity over the whole time of motion of the body, and the
instantaneous power of gravitational force as a function of time.
Solution: Average power of gravity over the whole time of motion can be given as
Total work done by gravity
Average Power =
Total timeof motion
Now, for the complete motion, total displacement of the particle is equal to the horizontal range
R. Thus,
Total work done by gravity   mg    R   cos90o  0
So, AveragePower =0
Now we find the instantaneous power of gravity after a time t from time of projection when

particle has velocity v shown in figure:


v 

u
 mg
x

The velocity of the particle and force on the particle are given as:

v   u cos  iˆ   u sin   gt  ˆj

F  mg ˆj
Instantaneous power can be given as
 
P  F .v

  
P  mg ˆj . u cos iˆ  u sin  ˆj  gt ˆj 
 P  mg  gt  u sin  

Raman Niwas, Near Aakashvani, Mahmoorganj, Varanasi. Ph.(0542)–2363455,website www.catjee.in


[9]
Now Solve: Three balls are thrown with same speed as shown in figure from a tower of height h. Arrange
the average power developed by gravity until the balls come to ground in ascending order.

2
3
1

Ans: 2  3 1.
Example 7: An advertisement claims that a certain 1200 kg car can accelerate from rest to a speed of 25 m/s
in a time of 8s. What average power must the motor produce to cause this acceleration? (Ignore
friction losses)
Solution: The force required for the given acceleration is,
 25m/s 
F  1200kg      3570N
 8s 
And the total displacement during this time interval is:

 25  100m
2
v2
s 
2a 2  25/ 8 
Total work done in accelerating the car is given by
W = K =  3750N 100m  375kJ
Thus the average Power P is,
375kJ
Power =  46.9kW
8s
Now Solve: A scooter company gives the following specifications about its product.
Weight of the scooter —95 kg
Maximum speed — 60 km/h
Maximum engine power — 3.5hp
Pick up time to get maximum speed — 5s
Check the validity of these specifications.

Raman Niwas, Near Aakashvani, Mahmoorganj, Varanasi. Ph.(0542)–2363455,website www.catjee.in


[10]
IN CHAPTER EXERCISE  1

1. State the conditions under which a force performs zero work.


2. State carefully, if the following quantities are positive or negative:
(i) Work done by a man in lifting a bucket out of a well by means of a rope tied to the bucket,
(ii) Work done by gravitation force in the above case,
(iii) Work done by friction on a body sliding down an inclined plane,
(iv) Work done by the resistive force of air on vibrating pendulum in bringing it to rest.
3. What is the amount of work done by a force, when a body moves in a circular path with constant speed?
4. A particle of mass 20g is thrown vertically upwards with a speed of 10 m/s. Find the work done by the
force of gravity during the time the particle goes up.
     
5. A particle moves from a point r1   2m  i  3m  j to another point r2  3m  i   2m  j during which
  
a certain force F   5N  i  5N  j acts on it. Find the work done by the force on the particle during the
displacement.
6.  
A body is constrained to move in the y-direction. It is subjected to a force 2iˆ  15 ˆj  6kˆ N. What is
the work done by this force in moving the body through a distance of 10m in positive y direction?
7. A rigid body mass 2 kg initially at rest moves under the action of an applied horizontal force of 7 N on
a table with coefficient of kinetic friction = 0.1. Calculate the
(a) work done by the applied force on the body in 10s.
(b) work done by friction on the body in 10s.
(c) work done by the net force on the body in 10s.
(d) change in kinetic energy of the body in 10s.
8. A boy pulls a 5 kg block 20 m along a horizontal surface at a constant speed with a force directed 45°
above the horizontal. If the coefficient of kinetic friction is 0.2, how much work does the boy do on the
block?
9. An inclined plane is moving up with constant velocity v. A block kept on incline is at rest. Calculate the
work done by gravity, friction force, and normal reaction on block in time interval of t.

 v
m



10. A block of mass m is placed on the block of mass M as shown in figure. The horizontal force F acts on M
during time interval t. If the horizontal surface is smooth, assuming no relative sliding between the blocks,
find the
m
F
M

(i) Total work done by friction on both the blocks



(ii) Work done by F on the lower block

Raman Niwas, Near Aakashvani, Mahmoorganj, Varanasi. Ph.(0542)–2363455,website www.catjee.in


[11]
11. Spring A and B are identical except that A is stiffer than B i.e., force constant k A  kB . In which spring
is more work done; if
(i) they are stretched by the same amount?
(ii) they are stretched by the same force?
12. A force varying with distance is given as F  aebx acts on a particle of mass m moving in a straight
line. Find the work done on the particle in its displacement from origin to a distance d.
13. A block of mass 2.0 kg is pushed down an inclined plane of inclination 37° with a force of F  20 N
acting parallel to the incline. It is found that the block moves down the incline with an acceleration of
10 ms–2. If the block started from rest, find the work done
(i) by the applied force in the first second
(ii) by the weight of the block in the first second
(iii) by the frictional force acting on the block in the first second
14. A block of mass m is kept on a rough plank which moves with a horizontal acceleration a. If the plank
was at rest at t  0 , and the block does not slide relative to the plank, find the work done by friction on
the
(i) block (ii) plane, (iii) system (block + plank), during time t.
15. A block of mass m1 moves with an acceleration a12 relative to a trolley as shown in figure. The block is
being observed by two observers (2) and (3). The observer (2) is at rest with respect to trolley which is
moving with acceleration a2 while the observer (3) is moving on ground with acceleration a3 . What is
the work done by the pseudo force as observed by the observers (2) and (3) on the block during time t ?
Assume zero initial velocities of the bodies and observers.
2
a12
3
a2
Trolley
a3

16. A smooth block of mass m moves up from bottom to top of a wedge which is moving with an
acceleration a0 . Find the work done by the pseudo force measured by the person sitting at the edge of
the wedge.

h
a0

l
17. A force of 20.0 N is required to hold a spring stretched by 5.0 cm from its equilibrium position. How
much work was done in stretching the spring?
18. A body is thrown on a rough surface such that friction force acting on it is linearly varying with
distance travelled by it as f  ax  b. Find the work done by the friction on the box if before coming to
rest the box travels a distance s.
19. An electric motor that can develop 1.0 hp is used to lift a mass of 25 kg through a distance of 10.0 m.
What is the minimum time in which it can do this? (Take g = 10 m/s2)
Raman Niwas, Near Aakashvani, Mahmoorganj, Varanasi. Ph.(0542)–2363455,website www.catjee.in
[12]
20. A pump is required to lift 1000 kg of water per minute from a well 12 m deep and eject it with a speed
of 20 m/s. How much work is done per minute in lifting the water and what must be the power output of
the pump? (Take g = 10 m/s2)
21. A ball is released from the top of a tower. The ratio of work done by force of gravity in first, second and
third second of the motion of ball is:
(A) 1:2:3 (B) 1:4:16 (C) 1:3 :5 (D) 1:9:25
22. An object of mass m is tied to string of length L and a variable horizontal force is applied on it which
starts at zero and gradually increases (it is pulled extremely slowly so that equilibrium exists at all
times) until the string makes an angle  with the vertical. Work done by the force F is:

 L

F
m

(A) mgL(1  cos ) (B) FL sin  (C) mgL (D) FL(1  tan  )
  
23. A force F  6 xiˆ  2 yjˆ displaces a body from r1  3iˆ  8 ˆj to r2  5iˆ  4 ˆj. Find the work done by the
force.
24. From figure, find the work done at the end of displacements: (i) 20 cm, (ii) 40 cm, and (iii) 80 cm.
F (in N)
10

M N P R D
O x (in
10 20 30 40 50 60 70 80 cm)

–5

–10

25. A body moves from point A to B under the action of a force, varying in magnitude as shown in figure.
Obtain the work done. Force is expressed in newton and displacement in meter.
F(N)
Q
15 A P
10
5 R
0 s(m)
–5 1 2 3 4 5
–10
–15 B

Raman Niwas, Near Aakashvani, Mahmoorganj, Varanasi. Ph.(0542)–2363455,website www.catjee.in


[13]
26. A 10-kg block moves in a straight line under the action of a force that varies with position as shown in
figure. How much work does the force do as the block moves from origin to x  8m ?

10

Force (N)
5
Position (m)
0
2 4 6 8
–5

27. A particle of mass m is projected with velocity u at an angle  with horizontal. During the period when

the particle descends from highest point to the position where its velocity vector makes an angle with
2
horizontal. Work done by gravity force is:
1 2 2 1 2  1 2  1 2 
(A) mu tan  (B) mu tan 2 (C) mu cos 2  tan 2 (D) mu cos 2 sin 2 
2 2 2 2 2 2 2
28. A block of 1 kg is kept on a rough inclined surface of inclination 37o in an elevator moving up with
constant velocity of 5 m/s. In 10 second work done by normal reaction (no sliding on incline surface)
(i) from ground frame is 320 J
(ii) is equal to work done by friction force in elevator frame
(iii) is equal to work done by friction in ground frame
(A) (i) (B) (ii), (iii) (C) (i), (ii) (D) only (iii)
29. A small block of mass m is kept on a rough inclined surface of inclination  fixed in an elevator. The
elevator goes up with a uniform velocity v and the block does not slide on the wedge. The work done by
the force of friction on the block in time t will be:
(A) Zero (B) mgvt cos2  (C) mgvt sin 2  (D) mgvt sin 2
  yiˆ  xjˆ
30. A force F  b 2 N (b is constant) acts on a particle as it undergoes counterclockwise circular
x  y2
motion in a circle: x2  y 2  16. The work done by the force when the particle undergoes one complete
revolution is ( x, y are in m)
(A) Zero (B) 2 b J (C) 2b J (D) None of these
31. Under the action of a force, a 2 kg body moves such that its position x as a function of time is given by
x  t 3 / 3 where x is in metre and t in second. The work done by the force in the first two second is:
(A) 1600 J (B) 160 J (C) 16 J (D) 1.6 J
32. A spring, which is initially in its un-stretched condition, is first stretched by a length x and then again
by a further length x. The work done in the first case is W1 and in the second case is W2 :

(A) W2  W1 (B) W2  2W1 (C) W2  3W1 (D) W2  4W1

33. A uniform chain of length L and mass M is lying on a smooth table and one third of its length is
hanging vertically down over the edge of the table. If g is acceleration due to gravity, the work required
to pull the hanging part on to the table is:
mgL mgL mgL
(A) mgL (B) (C) (D)
3 9 18

Raman Niwas, Near Aakashvani, Mahmoorganj, Varanasi. Ph.(0542)–2363455,website www.catjee.in


[14]
34. Two equal masses are attached to the two ends of a spring constant k. The masses are pulled out
symmetrically to stretch the spring by a length x over its natural length. The work done by the spring on
each mass is:
1 2 1 1 1
(A) kx (B)  k x 2 (C) k x 2 (D)  k x 2
2 2 4 4
35. A particle of mass 0.1 kg is subjected to a force which varies with distance as shown in figure. If it
starts its journey from rest at x  0 , its velocity at x  12 m is:
F
10

x (m)
0
4 8 12
(A) 0 m/s (B) 20 2m/s (C) 20 3m/s (D) 40 m/s
36. Velocity-time graph of a particle moving in a straight line is as shown in figure. Mass of the particle is
2 kg. Work done by all the forces acting on the particle in time interval between t  0 to t  10 s is:
v (m/s)

10

10
–20 t (s)

(A) 300 J (B) – 300 J (C) 400 J (D) – 400 J


37. The following figure illustrates the relation between the position (x) of a particle and force. The work
done in displacing the particle from x  1 to 5 m is:

20
Force in N

10
0
1 2 3 4 5 x in m
–10
–20

(A) 20 J (B) 60 J (C) 70 J (D) 100 J


38. A particle of mass m moves from rest under the action of a constant force F which acts for two seconds.
The maximum power attained is:
F2 2F 2F 2
(A) 2Fm (B) (C) (D)
m m m
39. A boy whose mass is 30 kg climbs, with a constant speed, a vertical rope of 6 m long in 10 s. The
power of the boy during the climb is: (Take g = 10 ms–2)
(A) 60 W (B) 3000 W (C) 180 W (D) 5 W

Raman Niwas, Near Aakashvani, Mahmoorganj, Varanasi. Ph.(0542)–2363455,website www.catjee.in


[15]
40. A particle of mass m is moving in a circular path of constant radius r such that its centripetal
acceleration ac is varying with time as ac  k 2 rt 2 , where k is a constant. The power delivered to the
particle by the force acting on it is:
1 4 25
(A) 2 mk 2 r 2t (B) mk 2 r 2t (C)mk r t (D) 0
3
41. Water is pumped from a depth of 10 m and delivered through a pipe of cross section 102 m2 upto a
height of 10 m. If it is needed to deliver a volume 0.2 m3 per second the power required will be:
(A) 19.6 kW (B) 9.8 kW (C) 39.2 kW (D) 4.9 kW
42. A block of mass m slides down a rough inclined plane of inclination  with horizontal with zero initial
velocity. The coefficient of friction between the block and the plane is  with   tan 1 ( ). Magnitude
of rate of work done by the force of friction at time t is:
(A)  mg 2t sin  (B) mg 2t (sin    cos ) (C)  mg 2t cos (sin    cos ) (D)  mg 2t cos

43. Select the correct alternative(s):


(A) Work done by static friction is always zero
(B) Work done by kinetic friction can be positive
(C) Kinetic energy of a system can not be increased without applying any external force on the system
(D) Work energy theorem is valid in non-inertial frames if we account pseudo force acting on the body
in the non-inertial frame
44. Work done by a force on an object is zero, if:
(A) The force is always perpendicular to its acceleration
(B) The object is stationary but the point of application of the force moves on the object
(C) The force is always perpendicular to its velocity
(D) The object moves in such a way that the point of application of the force remains fixed
45. In projectile motion power of the gravitational force:
(A) Is constant throughout
(B) Is negative for first half, zero at topmost point and positive for rest half
(C) Varies linearly with time
(D) Is positive for complete path

46. A person applies a constant force F on a particle of mass m and finds that the particle moves in a circle
of radius r with a uniform speed v as seen (in the plane of motion) from an inertial frame of reference.
Select the correct statement.
(A) This is not possible
(B) There are other forces on the particle

(C) The resultant of the other forces is mv 2 / r towards the center


(D) The resultant of the other forces varies in magnitude as well as in direction

Raman Niwas, Near Aakashvani, Mahmoorganj, Varanasi. Ph.(0542)–2363455,website www.catjee.in


[16]
ANSWER KEY

1. 20. [ 5333.33J/s ]
2. (i) positive (ii) negative 21. (C)
(iii) negative (iv) negative 22. (A)
3. Zero 23. 0J
4. –1.0 J 24. [(i) 1 J; (ii) 3J; (iii) 3.5 J]
5. Zero 25. 22.5 J
6. 150 J 26. 25 J
7. (i) 875 J, (ii) –250 J, (iii) 625 J, (iv) 625 J 27. (C)
8. [163.3 J] 28. (C)
9. [(i) mg vt (ii) mgvt cos2  (iii) mgvt sin 2  ] 29. (C)
30. (B)
F 2t 2 31. (C)
10. [(i) 0 (ii) ]
2 M  m 32. (C)
11. [(i) A (ii) A] 33. (D)
34. (D)
12. [
a
b
1  ebd  ] 35. (D)
13. [(i) 100 J, (ii) 60 J, (iii) –60 J] 36. (A)
37. (A)
1 1
14. [(i) ma 2t 2 , (ii) – ma 2t 2 , (iii) 0] 38. (D)
2 2
39. (C)
1 ma
15. [  ma2  a12t 2 ;  3 (a12  a2  a3 )t 2 ] 40. (B)
2 2
41. (C)
16. [ ma0l ]
42. (C)
17. [0.5 J] 43. (B,D)
1
18. [ as 2  bs ] 44. (B,C,D)
2 45. (B,C)
19. [ 3.35s ] 46. (B,D)

4. Energy
Energy is described as a quantity that an object or system possesses. Work is something that is done on
objects while energy is something that objects have.
Consider an object at rest on a frictionless surface. Let a horizontal force acts on the object which sets
the object in motion. In this process, work is done on the object. We consider the work is being ―used‖
in setting the body in motion or changing the kinetic condition of the body. Now since the body is set in
motion, we say the body has energy – Kinetic Energy, which gives it capability to do work. Energy of
an object is ability to do work. An object achieves an ability to do work, when some work is done on
object itself. Thus energy has same dimensional formula and SI unit as that of work.
Energy is of two kinds: Kinetic energy and Potential energy. Kinetic energy is the energy that particle
possess by virtue of its motion while Potential energy of a system is associated with the relative
positions of two or more constituent particles of the system. The importance of energy of a system lies
due to the property of its conservation in various processes.

Raman Niwas, Near Aakashvani, Mahmoorganj, Varanasi. Ph.(0542)–2363455,website www.catjee.in


[17]
5. Work Energy Theorem for a particle

Suppose a particle of mass m moves under the action of a certain force F , which is resultant of all

forces acting on the particle. During an elemental displacement ds , work performed dW by this force
is:
 
dW  F  ds

dv 
 dW  m  ds
dt
 
  
dW  m v  dv
 
Here it is to be noted that dv represents change in velocity vector while v represents the velocity vector
  
itself. The quantities dv and v need not to be in same directions. The direction of dv is along the

acceleration of the particle which may be at any angle to the velocity vector v . Now,
 
 dW  m v dv cos
 
Here dv cos can be considered as the component of change in velocity dv in the direction of velocity
 
vector v . Thus dv cos represents change in magnitude of velocity dv while the particle was moving

with speed v. Thus we get,

 dW  m  v  dv 
Integrating both sides with proper limit,
dW   m  v  dv 
W v2
 0 v1

W  m  v22  v12 
1

2

It is seen from this that work of the resultant force F contributes to an increase in a certain quantity,
referred to as Kinetic Energy. Hence, if K represents change in kinetic energy of the particle,
W  K
This is the mathematical equation for work energy theorem. In words,
the increment of kinetic energy gained by a particle over a certain displacement is equal to the
algebraic sum of works performed by all forces acting on the particle over the same displacement.
Also rate of work done by net force will then be equal to rate of increase of kinetic energy for the
particle.
Example 8: A particle of mass m is hanging with the help of string of length  . If air flow applied constant
mg
force towards right, then what is maximum angle deflected by string?
2
Solution: Figure shows the corresponding string:

Raman Niwas, Near Aakashvani, Mahmoorganj, Varanasi. Ph.(0542)–2363455,website www.catjee.in


[18]


l

Fair

By work energy theorem,


Wmg  Wair  WT  KE f  KEi

mg 
 mg  1  cos   sin   0  0  0
2
sin 
 cos   1    53
2
Now Solve: A particle of mass m is hanging with the help of string of length  inside a car on a horizontal
surface. If the car starts to accelerate horizontally with constant acceleration of g, find the
maximum angle deflected by string.
Ans: 90o
Example 9: A 40 kg dog jumps from a height of 2m onto a platform mounted on springs. As the spring
compress, the platform is pushed down a maximum distance of 0.4 m below its initial position,
and then it rebounds. Assuming platform to be light, what is the dog's speed at the instant the
platform is depressed to 0.2m ?
Solution:
dog

2m

dog 0.2 m
0.4 m
dog
Natural v=?
length
v=0

State-1 State-2 State-3


By work energy theorem from state 1 to state 2, Wg  Ws  KE f  KE1
1
 40  10  2.4  K (0.4)2  0  0
2
 K  12000 N/m
By work energy theorem from state (2) to state 3,
1 1
 40 10  0.2  K[ 0.4    0.2  ]  40 v 2  0
2 2

2 2
 v  4 2 m/s
Now Solve: In the previous problem find out the maximum height attained by the dog.
Ans: 2 m.

Raman Niwas, Near Aakashvani, Mahmoorganj, Varanasi. Ph.(0542)–2363455,website www.catjee.in


[19]
6. Work energy theorem for a system of particles

Let us consider a system of N particles such that ith particle initially has kinetic energy Ki. In a process,
if work done on ith particle due to all forces on ith particle is Wi, work energy theorem for this particle
gives,
Wi  Ki

 Wi (Internal)  Wi (External)  Ki

Here Wi (Internal) and Wi (External) represent the respective works performed by the internal forces of the
system on the ith particle and the external forces of the system on the ith particle. The above equation
holds for all individual particles of the system, and collection of such equation for each particle of the
system collectively gives:
WTotal Internal  WTotal External   Ki
i

 WTotal Internal  WTotal External   KTotal

Thus, increment in kinetic energy of a system is equal to the work performed by all the forces acting on
all the particles of that system. This ―work performed by all the forces‖ includes, internal forces acting
within the system as well as external forces acting on the system (it also includes the Pseudo force, if
the considered frame of reference is non-inertial).
Example 10: A truck is moving with constant acceleration a0. A block of mass m is kept on the rough trolley
of the truck and is observed to remain stationary with respect to truck. Using work energy
theorem, find the velocity of block:
(i) relative to ground, and
(ii) relative to truck, when truck moves a distance x from the starting point.
Solution: (i) From the ground frame of reference, since there is no relative motion between the block
and the truck, therefore the static force of friction on the block is f = ma in forward
direction as shown in the figure
N

f = ma0
mg

If the truck moves a distance x on the ground, the block will also move the same distance x as
there is no slipping between the two. Hence, work done by friction on the block (w.r.t. ground)
is
W f   ma0  x 
If the speed of the block relative to ground after travelling the distance x is v , then from work
energy theorem,
1 2 1 2
 mv    m  0    ma0 x
2  2 
 v  2a0 x

Raman Niwas, Near Aakashvani, Mahmoorganj, Varanasi. Ph.(0542)–2363455,website www.catjee.in


[20]
(ii) Relative to the accelerated truck frame, following is the free body diagram:
N

FPseudo
f = ma0
mg

Since the considered frame of reference is non-inertial, the work performed by Pseudo
force must also be considered while applying work energy theorem. Thus, if vr
represents the velocity of block relative to truck,
1 2 1 2
 mvr    m  0     ma0 x    ma0 x 
2  2 
 vr  0
Thus the block will remain stationary relative to the truck.
Now Solve: A truck is moving with constant acceleration a0. A block of mass m is kept on the rough trolley
of the truck and is observed to be moving with respect to truck. Using work energy theorem,
find the velocity of block:
(i) relative to ground, and
(ii) relative to truck, when truck moves a distance x from the starting point.
Ans: (i)
(ii)
Example 11: A block of mass m  4kg is dragged 2m, along a horizontal surface by a force F  30 N acting
at an angle 53° to the horizontal. The initial speed is 3 m/s and k  1/ 8.
F
53°

(i) Find the change in kinetic energy of the block


(ii) Find its final speed
Solution (i) The force acting on the block are shown in the figure.
N
F

f 53°

mg

Clearly, works performed by Normal force and the gravitational force are zero.
Whereas work performed by forces F and f are:
WF  Fs cos53o
and W f   fs  k Ns where N  mg  F sin 53o
From work energy theorem,
K  Wnet  WF  W f
Therefore, by substituting the given values we get:
K  F s cosθ  k  mg  f sin   s

Raman Niwas, Near Aakashvani, Mahmoorganj, Varanasi. Ph.(0542)–2363455,website www.catjee.in


[21]
1
 K   30  2  0.6    40  24  2  32 J
8
1 1
(b) Now K  mv 2f  mvi2  32 J
2 2
Since vi  3m/s.
Therefore, v f  5m/s
Now Solve: Solve the previous problem if the block is dragged by a force F  59 N as shown in figure.
F

53°
Ans: 66 J
Example 12: A box of mass m is gently placed on a conveyor belt that moves at a constant speed v. The
coefficient of kinetic friction is k .
m v

(i) What is the work done by friction till slipping stops?


(ii) How far does the box move before reaching its final speed?
Solution: (i) When the box is first placed on the belt there will be slipping between the two. But the
force of friction on the box and its displacement are in the same direction.
N

mg

Consequently, the work done by kinetic friction is positive. Taking the final speed of
the box to be v, work performed by friction can be written as:
1
W f  K   mv 2
2
(ii) The force of friction is
f  k N  k mg

If total displacement of the block relative to ground is d,


1
 k mgd   mv 2
2

v2
 d
2k g

Now Solve: Solve the previous problem with respect to belt.

Raman Niwas, Near Aakashvani, Mahmoorganj, Varanasi. Ph.(0542)–2363455,website www.catjee.in


[22]
Example 13: In the figure shown, all surfaces are smooth and force constant of spring is 10 N/m. Block of
mass 2 kg is not attached with the spring. The spring is compressed by 2m and then released.
Find the maximum distance ‗d‘ traveled by the block over the inclined plane. Take g  10 m/s2.

10 N/m 2 kg
30

2m
Solution: In the final position, block will stop for a moment and then it will return back. In the initial
1
position system has only spring potential energy kx 2 and in the final position it has only
2
gravitational potential energy.
v=0
Final
Position
d h
30
Initial
Position
Since, all surfaces are smooth, therefore mechanical energy will remain conserved.
1 2 d 
 Ei  E f or kx  mgh  mg  
2 2
d
Where h  d sin 30 
2
2
kx
 d
mg
Substituting the values we have,
10  2 
2

d =2m
 2 10 
Now Solve: Solve the previous problem if the inclined plain is rough and coefficient of friction is 0.2.
Example 14: A smooth narrow tube in the form of an arc AB of a circle of centre O and radius r is fixed so
that A is vertically above O and OB is horizontal. Particles P of mass m and Q of mass 2 m with
a light inextensible string of length (  r / 2) connecting them are placed inside the tube with P
and A and Q at B and released from rest. Assuming the string remains taut during motion, find
the speed of particles when P reaches B.

A
P

O B
Q
Solution: All surfaces are smooth. Therefore, mechanical energy of the system will remain conserved.
 Decrease in PE of both the blocks = increase in KE of both the blocks
r  1
  mgr    2mg      m  2m  v
2

 2  2
2
or v 1    gr
3
Raman Niwas, Near Aakashvani, Mahmoorganj, Varanasi. Ph.(0542)–2363455,website www.catjee.in
[23]
Now Solve: A smooth narrow tube in the form of an arc AB of a circle of centre O and radius r is fixed so
that A is vertically above O and OB is horizontal. Particles P of mass m, R of mass 4m and Q of
mass 2 m with a light inextensible string of length (  r / 2) connecting them are placed inside
the tube with P at A, R at C and Q at B as shown in figure are released from rest. Assuming the
string remains taut during motion, find the speed of particles when P reaches B.

A
P C
R

30°
O B
Q
gr
Ans: 3  2 
4 

7. Types of forces
While analysing the problems using the principle of conservation of energy, it is important to
distinguish between two types of forces:
7.1 Conservative Forces: A force is conservative if the work done by the force on a particle is
independent of the path followed by the particle. Work done by such forces on the object
depends only on the initial and final positions of the object. The net work done by such forces
in moving an object through any closed path is zero. Examples of conservative forces are
Gravitational force, ideal spring forces and electrostatic forces.
7.2 Non-conservative forces: Those forces which do not satisfy the above mentioned criteria are
not conservative in nature. Friction and viscous forces are the most common examples of non-
conservative forces.
Following are few basic properties of conservative forces and non-conservative forces, based on which
these forces can be identified:
(i) All constant forces are conservative forces.
(ii) All forces which are dependent on magnitude or direction of instantaneous velocity are
non-conservative forces.
(iii) If work performed by a force while the object moves along the loop is non-zero, then
the force is non-conservative
Example 15:  
A body is displaced from origin to (1m, 1m) by a force F  2 yiˆ  3x 2 ˆj along two paths

(i) x  y (ii) y  x 2

Find the work done along both paths separately.

Solution: Here, 
F  2 yiˆ  3x 2 ˆj  and 
dr  dxiˆ  dyjˆ 
 
Thus, dW  F  dr   2 ydx  3x 2 dy 

Raman Niwas, Near Aakashvani, Mahmoorganj, Varanasi. Ph.(0542)–2363455,website www.catjee.in


[24]

 
We cannot integrate 2 ydx  3x dy as such to find the work done. But along the given paths
2

we can change the expression.


(i) Along the path x  y,

dW   2 ydx  3x2 dy    2 xdx  3 y 2 dy 


1m ,1m 

  2 xdx  3 y dy    x
1m ,1m 
W1   y 3   1  1  2 J
2 2 2 3
 0,0
 0,0 

(ii) Along the path y  x 2

dW   2 ydx  3x2 dy    2 x2 dx  3 ydy 


1m,1m  1m,1m  1m ,1m 
2 3 
   2 3 3 2 13
W2   F  dr  2 x dx  3 ydy =  x3  y 2 
2
 1  1  J
 0,0   0,0  3 2   0,0 3 2 6

Note: We can see that W1  W2 or work done is path dependent in this case.

Now Solve: Solve the previous problem if F  xiˆ  y ˆj
Ans: 1 J in both cases.

Note: We can see that W1  W2 or work done is path independent in this case.

8. Potential energy
This is the type of energy which is possessed by a system due to relative separation of particles of the
system. It can be understood from the following analysis under two categories:
8.1 Potential energy of System: Consider an isolated system on which no external force applying
agent performs any work. Under such case, the change in kinetic energy of the system will be
caused only due to works performed by internal forces. Thus for such systems,
WTotal  KE
 WConservative  WNon-Conservative  KE
In the absence of works performed by non-conservative forces within the system,
WConservative  KE
 KE  WConservative  0
Here WConservative represents the work performed by internal conservative force on the considered
system. From previous analysis we can say, this work depends on relative separation of the
particles and is independent of frame of reference.
Here we can define another kind of energy which is independent of frame of reference and
depends on relative separation of particles. This kind of energy is Potential Energy. Change in
potential energy for a system can be defined as negative of the work performed by conservative
internal forces within the system. Thus,
KE  PE  0
Note that this equation holds only if non-conservative forces perform no work in the system.
Potential energy can be considered as the type of energy that a system of particles has, due to
virtue of relative separation of the particles. Generally, potential energy of a system is
considered zero if all particles of the system are at infinite separation relative to each other (i.e.
the particles are not interacting with each other).
Raman Niwas, Near Aakashvani, Mahmoorganj, Varanasi. Ph.(0542)–2363455,website www.catjee.in
[25]
8.2 Potential energy of Particle: Consider a system of particles in which only one particle moves
relative to other particles. Work performed by internal conservative force on the moving
particle will cause change in potential energy of the system. Since the potential energy of the
system has been brought by change in position of only one particle, it is often convenient to
assign change in “potential energy of the system” as change in “potential energy of the
particle”. Following are few properties of potential energy of a particle:
(i) It is generally considered zero when particle is not interacting with any other particle of
the system.
(ii) Its absolute values can be considered zero at any point (called reference point).
(iii) Change in potential energy only depends on change in position of the considered particle.
Following are few examples from which we can further understand the meaning of
potential energy:
(i) If a ball is projected up, its kinetic energy decreases. This decrease in kinetic energy can
be accounted either from the negative work performed by the gravitational force or the
increase in potential energy on the ball. Here the potential energy associated with
gravitational force is called gravitational potential energy. This gravitational potential
energy is associated with the system consists of earth and the ball, but assuming earth to
remain stationary (why?) this potential energy can be assigned to the ball only.
v=0
Kinetic Energy minimum Potential Energy maximum

v0

Kinetic Energy maximum Potential Energy minimum

(ii) If two positive point charges are kept close to each other and released, both particles
will attain kinetic energy due to mutual repulsion between the charged particles. Such
increase in kinetic energy can be accounted either by considering positive work
performed by electrostatic repulsive force on the two charges or by decrease in
potential energy of system of two particles. Such potential energy caused due to
electrostatic interactions is called electrostatic potential energy. Note that this
electrostatic potential energy is associated strictly with system of the two particles and
not with either particle only.
Potential Energy maximum
& kinetic energy minimum
+ +

+ +
Potential Energy decreases and kinetic energy increases

(iii) If two blocks are connected together by a compressed spring and are released on a
smooth horizontal surface, the spring will apply force on the blocks and will increase
their respective kinetic energy. This increase in potential energy can be accounted on
the basis of positive work performed by spring or the decrease in potential energy of
the spring. Such potential energy associated with force due to spring is called spring

Raman Niwas, Near Aakashvani, Mahmoorganj, Varanasi. Ph.(0542)–2363455,website www.catjee.in


[26]
potential energy or elastic potential energy. This potential energy is considered to be
associated with the spring itself.
Compressed spring Potential Energy maximum
Kinetic Energy minimum

Natural Length
Potential Energy minimum
Kinetic Energy maximum

Example 16: A pump on the ground floor of a building can pump up water to fill a tank of value 30 m3 in 15
min. If the tank is 40 m above the ground and the efficiency of the pump is 30%, how much
electric power is consumed by the pump?
Solution: Mass of water to be pumped up,
m  Volume × density of water  30 103  3 104 kg
Height of the tank, h  40m
Work done by the pump to fill the tank,
W  mgh  3 104  9.8  40J  1.176 10 J
7

Time, t  15min  15  60s  900s


 Required average power,
W 1.176  107
P   13.07kW
t 900
Percentage efficiency of pump = 30%
Output power
Now %   100
Input power
Or Input power = power consumed by the pump
Output power 13.07
  100   100  43.55kW
% 30
 43.6kW
Example 17: A person trying to loose weight (dieter) lifts a 10 kg mass 0.5 m 1000 times. Assume that the
potential energy lost each time she lowers the mass is dissipated.
(a) How much work does she do against the gravitational force?
(b) Fat supplies 3.8 107 J of energy per kg, which is converted to mechanical energy with a
20% efficiency rate. How much fat will the dieter use up?
Solution: (a) Work done against gravitational force
 mngh
 10 1000  9.8  0.5
 49000J
(b) Energy supplied by fat per kg = 3.8 107 J
i.e. 3.8 107 J energy is equivalent to 1 kg of fat
 49 103 J energy will be equivalent to

Raman Niwas, Near Aakashvani, Mahmoorganj, Varanasi. Ph.(0542)–2363455,website www.catjee.in


[27]
40  103
 kg fat
3.8  107
49
Fat produced   104 kg
3.8
Fat produced
 %  20%  %   100
Fats used up bydieter

104 100
or 20  49  
3.8 Fats used up bydieter
or fats used up by the dieter
49  104 100
 
3.8 20
245
  104
3.8
245
  103 kg
38
 6.447 103 kg
 6.45 103 kg
9. Conservation of Energy

We have derived the relation:


KE  PE  0
This was true for a system of particles on which no work is performed by external forces and no
internal non-conservative forces perform work within the system. This relation can also be expressed
as:
 KE  f   PE    PE    0
  KE i  f i

  KE  f
  PE     KE    PE  
f i i

  KE    PE    KE    PE 
f i

Here subscript i and f represent initial and final values. Thus we can state principle of conservation of
mechanical energy for a system of particles as:
“If no work is performed by external force on a system of particles and no work is performed by
internal non-conservative forces within the system, then sum of kinetic and potential energy (called
mechanical energy) remains conserved for the system.”
While applying the principle of conservation of mechanical energy we may require results for
gravitational potential energy of a particle (assuming earth to be stationary) and spring potential energy.
These results are derived as below:
9.1 Gravitational Potential Energy: The work done by gravity on a particle of mass m whose
vertical height increases from y A to yB is
Wg  mg  h 
Thus the associated change in gravitational potential energy is:
PE  mg  h 

Raman Niwas, Near Aakashvani, Mahmoorganj, Varanasi. Ph.(0542)–2363455,website www.catjee.in


[28]
Any horizontal line can be considered as a reference of gravitational energy zero according to
convenience.
9.2 Spring Potential energy: Considering one end of the spring fixed (with spring constant k), if
other end moves such that elongation in the spring changes from x to x  dx , then work dW
done by the spring force is:
 
  
dW  k x  dx

Integration yields,

W   k  x 2f  xi2 
1
2
Thus the associated increase in spring potential energy is given by:

PE  k  x 2f  xi2 
1
2
Conventionally, spring potential energy is considered zero when elongation in the spring
remains zero. Thus the spring potential energy is:
1
PE  kx 2
2
Example 18: A block of mass m is pushed against a spring of spring constant k fixed at one end to a wall.
The block can slide on a frictionless table as shown in figure. The natural length of the spring is
L0 and it is compressed to half its natural length when the block is released. Find the velocity of
the block as a function of its distance x from the wall.
m
k
v

L0/2
x

Solution: When the block is released, the spring pushes it towards right. The velocity of the block
increases till the spring acquires its natural length. Thereafter, the block loses contact with the
spring and moves with constant velocity.
Initially, the compression of the spring is L0 / 2. When the distance of the block from the wall
becomes x, where x  L0 , the compression is ( L0  x). Using the principle of conservation of
energy,
2
1  L0  1 1
k    k  L0  x   mv 2 .
2

2  2 2 2
Solving this,
1/ 2
k  L20 
v   ( L0  x) 
m4 
k L0
When the spring acquires its natural length, x  L0 and v  . Thereafter, the block
m 2
continues with this velocity.

Raman Niwas, Near Aakashvani, Mahmoorganj, Varanasi. Ph.(0542)–2363455,website www.catjee.in


[29]
Example 19: A particle is placed at the point A of a frictionless track ABC as shown in figure. It is pushed
slightly towards right. Find its speed when it reaches the point B. Take g = 10 m/s2.
A C

1m B
0.5m

Solution: Let us take the gravitational potential energy to be zero at the horizontal surface shown in the
figure. The potential energies of the particle at A and B are
U A  Mg 1m 

and U B  Mg  0.5m .
The kinetic energy at the point A is zero. As the track is frictionless, no energy is lost. The
normal force on the particle does no work. Applying the principle of conservation of energy,
U A  K A  U B  KB
1
or Mg 1m  =Mg  0.5m   MvB2
2
1 2
or, vB  g 1m  0.5m 
2
 10m/s2   0.5m

 5m2 /s 2

or, vB  10 m/s.

Raman Niwas, Near Aakashvani, Mahmoorganj, Varanasi. Ph.(0542)–2363455,website www.catjee.in


[30]
IN CHAPTER EXERCISE2

47.

 
A 150 g mass has a velocity v = 2iˆ  6 ˆj m / s at a certain instant. What is its kinetic energy?

48. The momentum of a body is increased by 20%. Find the percentage increase in its kinetic energy.
49. A body dropped from a height H reaches the ground with a speed of 1.2 gH . Calculate the work done
by air-friction.
50. A block of mass 250 g slides down an incline of inclination 37° with a uniform speed. Find the work
done against the friction as the block slides through 1.0 m.
51. A block of mass m is kept over another block of mass M and the system rests on a horizontal surface
F
(fig.) A constant horizontal force F acting on the lower block produces an acceleration in
2m  M 
the system, the two blocks always move together. (a) Find the coefficient of kinetic friction between the
bigger block and the horizontal surface. (b) Find the frictional force acting on the smaller block. (c)
Find the work done by the force of friction on the smaller block by the bigger block during a
displacement d of the system.
m
M F

52. Find the average frictional force needed to stop a car weighing 500 kg in a distance of 25 m if the initial
speed is 72 km/h.
53. Find the average force needed to accelerate a car weighing 500 kg from rest to 72 km/h in a distance of
25 m.
54. A particle of mass m moves on a straight line with its velocity varying with the distance travelled
according to the equation v  a x , where a is a constant. Find the total work done by all the forces
during a displacement from x = 0 to x = d.
55. A 250 g block slides on a rough horizontal table. Find the work done by the frictional force in bringing
the block to rest if it is initially moving at a speed of 40 cm/s. If the friction coefficient between the
table and the block is 0.1, how far does the block move before coming to rest?
56. A block of mass 100 g is moved with a speed of 5.0 m/s at the highest point in a closed circular tube of
radius 10 cm kept in a vertical plane. The cross-section of the tube is such that the block just fits in it.
The block makes several oscillations inside the tube and finally stops at the lowest point. Find the work
done by the tube on the block during the process.
57. A car weighing 1400 kg is moving at a speed of 54 km/h up a hill when the motor stops. If it is just able
to reach the destination which is at a height of 10 m above the point, calculate the work done against
friction (negative of the work done by the friction).
58. Figure shows particle sliding on a frictionless track which terminates in a straight horizontal section. If
the particle starts slipping from the point A, how far away from the track will the particle hit the
ground?

A
1.0m
0.5m

Raman Niwas, Near Aakashvani, Mahmoorganj, Varanasi. Ph.(0542)–2363455,website www.catjee.in


[31]
–1
59. A bullet of mass 20 g is fired from a rifle with a velocity of 800 ms , After passing through a mud wall
100 cm thick, velocity drops to 100 m s–1. What is the average resistance of the wall ? (Neglect friction
due to air and work of gravity).
60. A body moving at 2 m/s can be stopped over a distance x. If its kinetic energy is doubled, how long will
it go before coming to rest, if the retarding force remains unchanged?
(A) x (B) 2x (C) 4x (D) 8x
61. A retarding force is applied to stop a train. The train stops after 80 m. If the speed is doubled, then the
distance travelled when same retarding force is applied is
(A) The same (B) Doubled (C) Halved (D) Four times
62. A particle moves in a straight line with retardation proportional to its displacement. Its loss of kinetic
energy for any displacement x is proportional to
(A) x2 (B) ex (C) x (D) logex
63. A block of mass 2 kg gently kept on the inclined conveyor belt as shown. Find work done by frictional
force on block in first 10 s. (Assuming belt to be long)
 = 0.8

2 m/s
2 m/s
37°

64. A uniform rope of mass M and length L is kept on smooth surface AB as shown. A horizontal constant
force F pulls the rope on rough surface, having co-efficient of friction .
F
A B Rough () C

(i) Find speed of the rope as it reaches completely on rough surface.


(ii) Find minimum force F, that can bring the entire rope on rough surface
65. A uniform chain of mass M and length L is hanging with half of it's length vertical. Find work done by
variable force is slowly bringing the rope completely on table.
F

66. A uniform chain is held on a frictionless table with one third of its length hanging over the edge. If the
chain has a length l and a mass m, how much work is required to pull the hanging part back on the
table?
67. The displacement of a body of mass 2 kg varies with time t as S  t 2  2t , where S is in meters and t is
in seconds. The work done by all the forces acting on the body during the time interval t  2 s to t  4s
is:
(A) 36 J (B) 64 J (C) 100 J (D) 120 J
Raman Niwas, Near Aakashvani, Mahmoorganj, Varanasi. Ph.(0542)–2363455,website www.catjee.in
[32]
68. A ball of mass m and density  is immersed in a liquid of density 3 at a depth h and released. To
what height will the ball jump up above the surface of liquid (neglect the resistance of water and air):
(A) h (B) h / 2 (C) 2h (D) 3h
69. A bob is suspended from a crane by a cable of length l  5 m. The crane and load are moving at a
constant speed v0 . The crane is stopped by a bumper and the bob on the cable swings out an angle of
60. The initial speed v0 is: (Take g = 9.8 m/s2)
l

v0

(A) 10 m/s (B) 7 m/s (C) 4 m/s (D) 2 m/s


70. A ball of mass m is thrown upward with a velocity v. If air exerts an average resisting force F, the
velocity with which the ball returns to the thrower is:
mg F mg  F mg  F
(A) v (B) v (C) v (D) v
mg  F mg  F mg  F mg  F
71. In the figure shown, the net work done by the tension when the bigger block of mass M touches the
ground is:

m
M
d

(A)  Mgd (B)   M  m  gd (C) Mgd (D) Zero

72. A stone is projected vertically up with a velocity u, reaches upto a maximum height h. When it is at a
height of 3h / 4 from the ground, the ratio of KE and PE at that point is: (consider PE  0 at the
point of projection)
(A) 1 : 1 (B) 1 : 2 (C) 1 : 3 (D) 3 : 1
73. A wedge of mass M fitted with a spring of stiffness ‗k‘ is kept on a smooth horizontal surface. A rod of
mass m is kept on the wedge as shown in the figure. System is in equilibrium and at rest assuming that
all surfaces are smooth, the potential energy stored in the spring as:

k m
M

mg 2 tan 2  m2 g tan 2  m2 g 2 tan 2  m2 g 2 tan 2 


(A) (B) (C) (D)
2K 2K 2K K

Raman Niwas, Near Aakashvani, Mahmoorganj, Varanasi. Ph.(0542)–2363455,website www.catjee.in


[33]
74. An ice cube of size a  10 cm is floating in a tank (base area A  50 cm  50 cm) partially filled with
water. The change in gravitational potential energy, when ice melts completely is: (density of ice is 900
kg/m3)
(A) –0.072 J (B) –0.24 J (C) –0.016 J (D) –0.045 J
75. A particle is released from a height H. At certain height its kinetic energy is two times its potential
energy with reference at bottom point. Height and speed of particle at that instant are:

H 2 gH H gH 2 H 2 gH H
(A) , (B) ,2 (C) , (D) , 2 gh
3 3 3 3 3 3 3
76. A uniform flexible chain of mass m and length 2l hangs in equilibrium over a smooth horizontal pin of
negligible diameter. One end of the chain is given a small vertical displacement so that the chain slips
over pin. The speed of the chain when it leaves pin is:

(A) 3gl (B) 2g (C) gl (D) 4g


77. Work done by the conservative forces in a system of particles is equal to:
(A) The change in kinetic energy of the system
(B) The change in potential energy of the system
(C) The change in total mechanical energy of the system
(D) None of these
78. System shown in figure is released from rest. Pulley and spring is massless and friction is absent
everywhere. The speed of 5 kg block when 2 kg block leaves the contact with ground is: (Take force
constant of spring k = 40 N/m and g = 10 m/s2)

5 kg
2 kg

(A) 2 m/s (B) 2 2 m/s (C) 2 m/s (D) 4 2 m/s


79. A 10 kg block is released from rest at the top of a incline and brought to rest momentarily after
compressing the spring by 2 metres. What is the speed of mass just before it reaches the spring:

10kg

k = 100 N/m

30
(A) 20 m/s (B) 30 m/s (C) 10 m/s (D) 40 m/s

Raman Niwas, Near Aakashvani, Mahmoorganj, Varanasi. Ph.(0542)–2363455,website www.catjee.in


[34]
80. A flexible chain of length L  20 2 m and weight W  10 kg is initially placed at rest on a smooth
frictionless wedge surface ABC. It is given a slight jerk on one – side so that it will start sliding on the
side. Find the speed of the chain when its one end will have the vertex of the wedge: (Take g = 10 m/s2)
B

A C

 
1/2
(A) 10 2 m/s (B) 10 m/s (C) 4 m/s (D) 10 2
81. If the block in the shown arrangement is acted upon by a constant force F for t  0, its maximum speed
will be:

k F
m

F 2F F 2F
(A) (B) (C) (D)
mk mk 2mk mk
82. A block of mass m is attached with a spring in its natural length, of spring constant k. The other end A
of spring is moved with a constant acceleration ' a ' away from the block as shown in figure. Find the
maximum extension in the spring. Assume that initially block and spring is at rest with respect to
ground frame:

A
m a

ma 1 ma 2ma 1 4ma
(A) (B) (C) (D)
k 2 k k 2 k

83. In the figure, the balls A is released from rest when the spring is at its natural (unstretched) length. For
the block B, of mass M to leave contact with the ground at some stage, the minimum mass of A must be:

B M

(A) 2M (B) M

(C) M / 2 (D) A function of M and the force constant of the spring.

Raman Niwas, Near Aakashvani, Mahmoorganj, Varanasi. Ph.(0542)–2363455,website www.catjee.in


[35]
84. A block Q of mass 2m is placed on a horizontal frictionless plane. A second block of mass m is placed
on it and is connected to a spring of spring constant K, the two block are pulled by distance A. Block Q
oscillates without slipping. The work done by the friction force on block Q when the spring regains its
natural length is:

K s
P

1 2 2 2 1 2 1 2
(A) KA (B) KA (C) KA (D) KA
3 3 2 4
85. A bead of mass 5 kg is free to slide on the horizontal rod AB. They are connected to two identical
springs of natural length h ms. as shown. If initial bead was at O & M is vertically below L then,
velocity of bead at point N will be:
L

37 K=1000 N/m


h
m
N O B

h
37 K=1000 N/m

M
(A) 5 h m/s (B) 40 h / 3 m/s (C) 8 h m/s (D) None of these
86. Velocity – time graph of a particle of mass 2 kg moving in a straight line is as shown in figure. Work
done by all the forces on the particle is:
v(m/s)
20

2 t(s)
(A) 400 J (B) –400 J (C) –200 J (D) 200 J

Raman Niwas, Near Aakashvani, Mahmoorganj, Varanasi. Ph.(0542)–2363455,website www.catjee.in


[36]
ANSWER KEY

47. 3J Mg
65.
48. (44%) 8
49. –0.28 mgH 66.
50. 1.5 J 67. [B]
F mF 68. [C]
51. (a) , (b) ,
2 M  m g 2 M  m
69. [B]
70. [C]
mFd 71. [D]
(c)
2 M  m 72. [C]
52. 4000 N 73. [C]
53. 4000 N 74. [D]
54. ma2 d/2 75. [B]
55. –0.02 J, 8.2 cm 76. [C]
56. –1.45 J 77. [A,B]
57. 20300 J 78. [B]
58. At a horizontal distance of 1 m from the 79. [A]
end of the track. 80. [B]
59. F = 6300 N 81. [A]
60. [B] 82. [C]
61. [D] 83. [C]
62. [A] 84. [A]
63. 184 J 85. [A]
86. [B]
2FL  Mg
64. (i)   gL (ii)
M 2

10. Application of Energy conservation in circular motion


For a particle to continue to perform circular motion, the force applying agents must be capable of
providing the required centripetal force. Here we will discuss the situations in which speed of the
revolving particle varies and hence requirement of centripetal force in various positions change. We
will see the requirements to be fulfilled for the particle to continue circular motion. Following two
categories describe circular motion of a particle subjected to Normal force and Tension force
separately:
10.1 Circular motions with normal force: The normal force is of pushing nature. Whenever a
surface is required to pull an object to keep the particle in a circle, the particle will leave the contact
from the surface instead. After the particle leaves the surface, it would continue to move as a projectile
(if gravitational force acts on the particle).
10.2 Circular motions with Tension force: The Tension force is of pulling nature. Whenever a
string is required to push a particle to keep the particle in a circle, the string will get slack instead. After
the string gets slack, the particle will continue to move as a projectile (if gravitational force acts on the
particle).

Raman Niwas, Near Aakashvani, Mahmoorganj, Varanasi. Ph.(0542)–2363455,website www.catjee.in


[37]
Example 20: In each of the following cases, identify whether the particle P will continue to perform circular
motion just after release (Assume each figure to represent vertical plane).

P v  2 gR v  4 gR
P
(i) (ii)
R

v  g  cos / 3
P v  2 gR
P
string
(iii) (iv) 
 string

Solution: Considering each particle P to perform circular motion, the required Normal force or Tension
force can be determined.
The free body diagrams of each particle, along with the required normal force or tension force
acting on the particle is simultaneously solved below:

 
2
P v  2 gR N 2gR
(i) mg  N  m
v2 R
R  N  mg
R

mg

 
2
4 gR
v  4 gR mg  N  m
(ii)
P v2 R
R mg  N  3mg

 
2
P v  2 gl 2gl
mg  T  m
(iii) v2 l
l mg  T  mg
 string

Raman Niwas, Near Aakashvani, Mahmoorganj, Varanasi. Ph.(0542)–2363455,website www.catjee.in


[38]
v  g  cos / 3 2
 gl cos  
 
2
v
P R  3 
string mg  mg cos   T  m
l
(iv) 
2
  T   mg
T 3

In parts (i) and (iv), the required Normal force and the Tension force are against the permissible
directions of their values. Thus in parts (i) and (iv) the contact will be lost and string will get slack
respectively.
Circular motion will continue only in parts (ii) and (iii).
Now Solve: If the speeds given to block/ball in the above four parts are:

3
(i) v  3gR (ii) v   0.5 gR (iii) v    gR (iv) v  2 gl cos 
4
respectively, then identify the cases in which circular motion will continue just after
projections.
Ans: In none of these cases.
Example 21: An insect starts slipping at the top of a spherical ball, with very small initial velocity, and
moves straight down the side. At what point, it will lose contact with the ball and fly of along
the tangent? That is, at the instant it loses contact with the ball, what angle does the radial line
from the centre to the insect makes with the vertical?
Solution: As it loses contact at B. Normal reaction will be zero at B.
So, at B only component of mg towards centre will give centripetal acceleration as Normal is
zero.

mv 2
mg cos 
r
A B

 v

By work energy theorem from A to B


1
mgr 1  cos   mv 2  0
2
2mg 1  cos   mg cos
2 2
 cos   cos 1  
3 3

Raman Niwas, Near Aakashvani, Mahmoorganj, Varanasi. Ph.(0542)–2363455,website www.catjee.in


[39]

Now Solve: Repeat the problem, if the insect was projected from top, horizontally with speed  0.5 gR .

4
Ans: cos 1  
3
Example 22: What is the minimum velocity imparted to a particle of mass m (which is hanging vertically
from a string of length 1) such that it completes vertical circle. Discuss what will happen if its
velocity is less than that or more than that value.
Solution: If we impart velocity v the particle starts its circular motion with decreasing speed because of
gravity. As the particle will go up, tension in string starts decreasing. So we have to give such a
velocity which will ensure non zero tension at every point. It can be achieved if tension at
highest point is non zero. Let at highest point, velocity is v1
X

m v
At highest point,
mv12 mv12
T  mg   T  mg
 
v1

m v

mv12
So,  mg  0 ... (i)

by work energy theorem from lowest to highest point
1 1 mv12 mv 2
mg 2  mv12  mv 2    4mg ... (ii)
2 2  
mv 2
By equation (i) and (ii),  5mg   v  5g

So minimum velocity to complete circle is 5g at lowest point

So if at lowest point velocity is less than 5g it will not complete the circle.

But if v  2 g  then particle can not, reach to the height of hinged point, it will oscillate in
lower half.
X

m v  2g 
Raman Niwas, Near Aakashvani, Mahmoorganj, Varanasi. Ph.(0542)–2363455,website www.catjee.in
[40]
If its velocity v  2 g  and v  5 g  then tension will be zero some where above hinge and
then particle will move freely in gravity
let v  4 g  then work energy theorem,
v1

 B

X T=0

A
v  2 g , 5g 
1 1
mg  1  cos   mv12  m  4 g   ... (i)
2 2
mv12
at B, F  mg  mg cos  0  ... (ii)

mg cos
 mg 1  cos    2mg q
2
2  2cos  cos  4
cos  2/ 3    cos1  2/ 3

It is the angle where tension becomes zero and particle follow projectile trajectory after that.

19
Now Solve: If initial speed of projection is gl , after covering what angle will the string become slack?
5
Ans: 127o
11. Relation between Potential energy and Force
We have earlier seen that the forces can be of two types: conservative and non-conservative. With each
conservative force, some potential energy can be assigned. In this section we will discuss the relation
between potential energy and the corresponding conservative force associated with this potential
energy.
11.1 Potential energy if force field is given: Consider a particle experiencing a conservative force
 
F and the associated potential energy be represented by U. Here F as well as U depends on
position co-ordinates x, y and z. While the particle moves from points A to B, increase in
potential energy of such a particle is defined as negative of work done by the corresponding
force. Mathematically,
B  

U    F  ds
A

If the points A and B are known, we first assign a path for the particle to follow. The path can
be any arbitrary path, but for simplicity we consider path to move first along x-axis, then along
y-axis and finally along z-axis. Integrations along these three parts of the path is then added to
find to total work performed by the force. Negative of such work gives the change in potential
energy. This is illustrated in following examples:

Raman Niwas, Near Aakashvani, Mahmoorganj, Varanasi. Ph.(0542)–2363455,website www.catjee.in


[41]
Example 23: A particle, which is constrained to move along x-axis, it subjected to a force in the same
direction which varies with the distance x of the particle from the origin as F  x   kx  ax3 .

Here, k and a are positive constants. Find potential energy as a function of x-coordinate of the
particle considering potential energy to be zero at origin.

Solution: We know that the potential energy of a particle is given by:


B
U    F  dx
A

Considering particle to start its motion from origin to x

U     kx  ax3  dx
x

kx 2 ax 4
 U  U At x 0  
2 4
kx 2 ax 4
 U 
2 4
This is the required potential energy function of the particle.
Now Solve: Repeat the problem considering force as a function of x-coordinate to be given by:
F  x   sin  x 

Also consider that potential energy is zero at x = 0.

Ans: cos  x   1

Example 24:  
A force F  k yiˆ  xjˆ (where k is a positive constant) acts on a particle moving in the x-y

plane. Find potential energy of the particle as a function of x and y coordinates assuming
potential energy of the particle to be zero at origin.
Solution: Let the particle first moves from origin O to point A (x, 0). And then from A to B (x, y). For the
displacement from O to A, the elemental displacement is

ds  dxi
 
Now, dW  F  ds
  
dW  k yiˆ  xjˆ  dxi   
A x ,0
 O
dW  k 
0,0
ydx
In RHS of the above equation, the y coordinate of the particle always remains zero. Thus
integration yields zero.
Thus, dWOA  0
Now for the path along A to B, the elemental displacement is

ds  dy j
 
Now, dW  F  ds
  
dW  k yiˆ  xjˆ  dy j   
B x, y
 A
dW  k 
x ,0
xdy

Raman Niwas, Near Aakashvani, Mahmoorganj, Varanasi. Ph.(0542)–2363455,website www.catjee.in


[42]
In RHS of the above equation, the x coordinate remains constant throughout the path. Hence
integration simplifies as,
B x, y
A
dW  kx 
x ,0
dy
 WAB  kxy
Thus the total work done by the conservative force from O to B is,
WTotal  WOA  WAB  kxy
And the corresponding potential energy is given as,
U  WTotal  kxy

Now Solve:   
A force F  k 1  y 2 iˆ  2 xyjˆ (where k is a positive constant) acts on a particle moving in
the x-y plane. Find potential energy of the particle as a function of x and y coordinates
assuming potential energy of the particle to be 10J at origin.
Ans: k  x  xy 2   10 in Joule.

11.2 Force if potential energy is given: Consider a particle experiencing a conservative force F

and the associated potential energy be represented by U. Here F as well as U depends on
position co-ordinates x, y and z. Let particle moves along x-axis by an elemental length dx.
During such displacement, only x-coordinate of the particle changes and change in potential is
dU. Then,

dU   F  dxi  
 
dU   Fx i  Fy j  Fz k  dxi  
 dU    Fx  dx
dU
 Fx  
dx Considering y & z to be constant
The above equation is symbolically written as:
U
Fx  
x
Thus x-component of the conservative force acting on the particle is the partial differentiation
(differentiation considering only x as variable) of potential energy with respect to x-coordinate.
Similarly, other components of the conservative force can also be written as:
U U
Fy   and Fz  
y z
In this manner, if the potential energy of the particle as function of co-ordinates are known, the
corresponding conservative force on the particle can be determined.
11.3 Force from graph of potential energy: For a particle free to move in straight line, the
conservative force acting on the particle is the negative of rate of change of potential energy of
the particle with respect to position. Thus slope of graph of potential energy v/s position
represents negative of the force. In the following graph:
potential energy (U)

A C

position of particle(x)

Raman Niwas, Near Aakashvani, Mahmoorganj, Varanasi. Ph.(0542)–2363455,website www.catjee.in


[43]
The slopes of the curve at A, B and C are positive, zero and negative respectively. Thus the
forces acting on the particle are along negative x-axis, zero and along positive x-axis
respectively.
Example 25: Potential energy U of a particle as a function of its x and y coordinates is given by:
U  kxy [Here k is positive constant]
Determine the corresponding conservative force acting on the particle as function of co-
ordinate.
Solution: We know that components of forces can be calculated along different directions using
following formula:
U U
Fx   and Fy  
x y

  kxy    kxy 
Thus, Fx   and Fy  
x y
 Fx    ky and Fy  kx

Thus the force vector is,




F  k yiˆ  xjˆ 
Note: The process is reverse of what we did in previous example.
Now Solve: Potential energy U of a particle as a function of its x and y coordinates is given by:
U  k  x  xy 2   10 [Here k is positive constant]
Determine the corresponding conservative force acting on the particle as function of co-
ordinate.
Ans: 
k 1  y 2  iˆ  2 xyjˆ 
12. Nature of Equilibrium
Equilibrium is state of a particle when acceleration of the particle is zero. If a particle is in state of
equilibrium as observed from an inertial frame of reference, then the net force acting on the particle is
zero.
If the particle is slightly disturbed from its state of equilibrium, the particle may or may not have
tendency regain its equilibrium state. Depending on behavior of the forces acting on the particle, after it
is slightly disturbed from its equilibrium state, following are the three types of equilibrium:
12.1 Stable Equilibrium: When a particle is slightly disturbed from its equilibrium state and then
the net force acts on the particle to again bring the particle in initial equilibrium state, then the
initial equilibrium of the particle is said to be stable equilibrium. Simplest example of such
equilibrium is a ball kept under equilibrium inside a concave surface as shown:

Ball in stable
equilibrium

If only conservative forces exist on the particle after it is displaced from equilibrium, then the
associated potential energy of the particle remains minimum in its stable equilibrium state.

Raman Niwas, Near Aakashvani, Mahmoorganj, Varanasi. Ph.(0542)–2363455,website www.catjee.in


[44]
12.2 Unstable equilibrium: When a particle is slightly disturbed from its equilibrium state and then
the net force acting on particle has no tendency to bring the particle in initial equilibrium state,
then the initial equilibrium of the particle is said to be unstable equilibrium. Simplest example
of such equilibrium is a ball kept under equilibrium on a convex surface as shown:

Ball in unstable
equilibrium

If only conservative forces exist on the particle after it is displaced from equilibrium, then the
associated potential energy of the particle remains maximum in its unstable equilibrium state.
12.3 Neutral equilibrium: When a particle is slightly displaced from equilibrium position and no
force acts on it then equilibrium is said to be neutral equilibrium
In the following potential energy graph for a particle free to move along x-axis, three different
types of equilibrium are shown:
Potential Energy
(U)

A B C

Position of particle(x)

The positions A, B and C are equilibrium positions (as slope of the curve is zero at these
positions.

Raman Niwas, Near Aakashvani, Mahmoorganj, Varanasi. Ph.(0542)–2363455,website www.catjee.in


[45]
INCHAPTER EXERCISE3

87. A 40 kg mass, hanging at the end of a rope of length l, oscillates in a vertical plane with an angular
amplitude  0 . What is the tension in the rope when it makes an angle  with the vertical? If the
breaking strength of the rope is 800 N, what is the maximum angular amplitude with which the mass
can oscillate without the rope breaking?
88. Figure shows a loop track of radius r. A box starts sliding from a platform at a distance h above the top
of the loop and goes around the loop without falling off the track. Find the minimum value of h for a
successful looping. Friction is negligible at all surfaces.

89. Figure shows a smooth track, a part of which is a circle of radius r. A block of mass m is pushed against
a spring of spring constant k fixed at the left end and is then released. Find the initial compression of the
spring so that the block presses the track with a force mg when it reaches the point P, where the radius
of the track is horizontal.

r
P
k

90. A particle is suspended from a fixed point by a string of length 5 m. It is projected from the equilibrium
position with such a velocity that the string stackns after the particle has reached a height 8 m above the
lowest point. Find the velocity of the particle, just before the string slackens. Find also, to what height
the particle can rise further.
91. A bob tied to the end of a string of length 2 m, other end of which is fixed at a point in vertical wall at a
point O. The bob is imparted a vertical downward velocity of 5 m/s when the string is horizontal and
swings in a vertical plane. Find the angular displacement of the bob from its initial position, when the
string breaks. Given that the tensile strength of the string is twice of the weight of the bob. Take g  10
m/s2.
92. In figure, a block slides along a track from one level to a higher level, by moving through an
intermediate valley. The track is frictionless until the block reaches the higher level. There a frictional
force stops the block in a distance d. The block‘s initial speed v0 is 6 m/s, the height difference h is 1.1
m and the coefficient of kinetic friction  is 0.6. Find d. Take g = 10 m/s2.
 = 0.6
v0 h
=0

Raman Niwas, Near Aakashvani, Mahmoorganj, Varanasi. Ph.(0542)–2363455,website www.catjee.in


[46]
93. A small particle slides along a track with elevated ends and a flat central part, as shown in figure. The
flat part has a length 3 m. The curved portions of the track are frictionless, but for the flat part the
coefficient of kinetic friction is  = 0.2. The particle is released at point A, which is at a height h = 1.5
m above the flat part of the track. Where does the particle finally come to rest? Take g = 10 m/s2.
A

3.0 m

94. A particle of mass 0.5 kg travels in a straight line with velocity v  ax3/2 where a  5m1/ 2s1. What is
the work done by the all force during its displacement from x = 0 to x = 2m?
95. The potential energy function of a particle in a region of space is given as U   2 xy  yz  J

Here x, y and z in metre. Find the force acting on the particle at a general point P  x, y, z .
96. The potential energy function for the force between two atoms in a diatomic molecule is approximately
a b
given by U  x   12  6 , where d are constant and x is the distance between the atoms. Find the
x x
dissociation energy of the molecule which is given as D  U  x   U at equilibrium  .
 

 x2 
97. The potential energy of a particle of mass 1 kg free to move along x-axis is given by U  x     x 
 2 
joule. If total mechanical energy of the particle is 2J, then find the maximum speed of the particle.
(Assuming only conservative force acts on particle)
98. Force between the atoms of a diatomic molecule has its origin in the interactions between the electrons
and the nuclei present in each atom. This force is conservative and associated potential energy U  r  is,
to a good approximation, represented by the Lennard – Jones potential.


 a 
12
a 
6

U  r   U 0      
 r 
 r 
Here r is the distance between the two atoms and U 0 and a are positive constants. Develop expression
for the associated force and find the equilibrium separation between the atoms.
99. A particle moves in the x-y plane in figure under the influence of a friction force with magnitude 3.00 N
and acting in the direction opposite to the particle‘s displacement. Calculate the work done by the
friction force on particle as it moves along the following closed paths: (a) path OA followed by the
return path AO, (b) path OA followed by AC and the return path CO, (c) path OC followed by the return
path CO, and (d) each of your three answers should be non-zero. What is the significance of this
observation?

Raman Niwas, Near Aakashvani, Mahmoorganj, Varanasi. Ph.(0542)–2363455,website www.catjee.in


[47]
y
C
B (5,00, 5.00)m

x
O A

100. A 4.00 – kg particle moves from the origin to position C, having coordinate x  5.00 m and y  5 m.
One force on the particle is the gravitational force acting in the negative y direction. Using equation
 
W  F r cos  F  r , calculate the work done by the gravitation force on the particle as it goes from
O at C along (a) OAC, (b) OBC, and (c) OC. Your results should all be identical. Why?
y
C
B (5,00, 5.00)m

x
O A

101.  
A force acting on a particle moving in the x–y plane is given by F  2 yiˆ  x 2 ˆj N, where x and y are in
meters. The particle moves from the origin to a final position having coordinates x  5.00 m as shown

in figure. Calculate the work done by F on the particle as it moves along (a) OAC, (b) OBC, and (c)

OC, (d) is F conservative or non – conservative? Explain.
y
C
B (5,00, 5.00)m

x
O A

102. A chain of length l and mass m lies of the surface of a smooth hemisphere of radius R  l with one end
tied to the top of the hemisphere. Taking mass of the hemisphere as reference line, find the gravitational
potential energy of the chain.

R d

R  d y = R cos
y

Raman Niwas, Near Aakashvani, Mahmoorganj, Varanasi. Ph.(0542)–2363455,website www.catjee.in


[48]
103. Under the action of a force, a 2 kg body moves such that its position x as a function of time is given by
x  t 3 / 3 where x in metre and t in second. The work done by the force in the first two second is:
(A) 1600 joule (B) 160 joule (C) 16 joule (D) 1.6 joule
104. A 1-kg stone at the end of 1m long string is whirled in a vertical circle at a constant speed of 4 ms –1.
The tension in the string is 6N when the stone is
(A) At the top of the circle (B) At the bottom of the circle
(C) Half way down (D) None of these
105. A stone of mass 1 kg tied to a light inextensible string of length L  10 / 3 m is whirling in a circular
path of radius L in a vertical plane. If the ratio of the maximum tension in string to the minimum
tension is 4 and if g is taken to be 10 ms–2, the speed of the stone at the highest point of the circle is
(A) 10 ms–1 (B) 5 3 ms–1 (C) 10 3 ms–1 (D) 20 ms–1
106. A ball of mass m is released from A inside a smooth wedge of mass m as shown in figure. What is the
speed of the wedge when the ball reaches point B?
A
R

45
B

Smooth
1/2
 5 gR 
1/ 2
 gR  3
(A)   (B) 2gR (C)   (D) gR
3 2  2 3 2

107. A small sphere is given vertical velocity of magnitude v0  5 ms–1 and it swings in a vertical plane
about the end of massless string. The angle  with the vertical at which string will break, knowing that
it can withstand a maximum tension equal to twice the weight of the sphere, is
l = 2m

v0

2 1
(A) cos 1 (B) cos 1   (C) 60 (D) 30
3 4
108. Two bodies of masses m and 4m are attached to a light string as shown in figure. A body of mass m
hanging from string is executing oscillations with angular amplitudes 60, while other body is at rest on
a horizontal surface. The minimum coefficient of friction between mass 4m and the horizontal surface is
(here pulley is light and smooth)
4m

1 3 1 1
(A) (B) (C) (D)
4 4 2 8

Raman Niwas, Near Aakashvani, Mahmoorganj, Varanasi. Ph.(0542)–2363455,website www.catjee.in


[49]
109. An object of mass m slides down a hill of height h of arbitrary shape and after travelling a certain
horizontal path stops because of friction. The friction coefficient is different for different segments for
the entire path but is independent of the velocity and direction of motion. The work that a force must
perform to return the object to its initial position along the same path is:
(A) mgh (B) 2 mgh (C) 4 mgh (D) – mgh
110. Two blocks of masses m1  1 kg and m2  2 kg are connected by non – deformed light spring. They are
lying on a rough horizontal surface. The coefficient of friction between the blocks and the surface is
0.4. What minimum constant force F has to be applied in horizontal direction to the block of mass m1
in order to shift the other block? (Take g = 10 m/s2)
F
m1 m2

(A) 8 N (B) 15 N (C) 10 N (D) 25 N


111. A block of mass m is attached with a massless spring of force constant k. The block is placed over a
3
rough inclined surface for which the coefficient of friction is   . The minimum value of M required
4
to move the block up the plane is (Neglect mass of string and pulley and friction in pulley):

M
m

37

3 4 3
(A) m (B) m (C) 2 m (D) m
5 5 2
112. A block of mass 5.0 kg slides down from the top of an inclined plane of length 3 m. The first 1 m of the
plane is smooth and the next 2 m is rough. The block is released from rest and again comes to rest at the
bottom of the plane. If the plane is inclined at 30 with the horizontal, find the coefficient of friction on
the rough portion.

P
Smooth
1m
2m
Rough
30
R

2 3 3 3
(A) (B) (C) (D)
3 2 4 5
113. A block of mass m has initial velocity u having direction towards +x axis. The block stops after
covering a distance S causing similar extension in the spring of constant K holding it. If  is the kinetic
friction between the block and the surface on which it was moving, the distance S is given by

m u

1 2 2 2 1
 mKu 2   2 m2 g 2 
1/2
(A)  mg (B)
K K
  mg   2 m2 g 2  mu 2 k
  m g  mK  2   mg 
1 2 2 2 1/2
(C) (D)
K k
Raman Niwas, Near Aakashvani, Mahmoorganj, Varanasi. Ph.(0542)–2363455,website www.catjee.in
[50]
114. In the figure shown, a spring of spring constant k is fixed at one end and the other end is attached to the
mass ‗m‘. The coefficient of friction between block and the inclined plane is ‗‘. The block is released
when the spring is in its natural length. Assuming that tan    , find the maximum speed of the block
during the motion.

m m
(A)  sin    cos  g (B)  sin    cos  g
k k
m m
(C)  cos   sin   g (D)  cos   sin   g
k k
115. A particle of mass 0.01 kg travels along a space curve with velocity given by 4iˆ  16kˆ m/s. After some
time, its velocity becomes 8iˆ  20 ˆj m/s due to the action of a conservative force. The work done on the
particle during this interval of time is:
(A) 0.32 J (B) 6.9 J (C) 9.6 J (D) 0.96 J
116. The potential energy of a particle of mass 1 kg in a conservative field is given as U   3x 2 y 2  6 x  J,
where x and y are measured in meter. Initially particle is at (1, 1) and at rest then:
(A) Initial acceleration of particle is 6 5 ms2
(B) Work done to slowly bring the particle to origin is 9 J
(C) Work done to slowly bring the particle to origin is –9J
(D) If particle is left free it moves in straight line
 
117. Given F   xy 2  iˆ   x 2 y  ˆj N. The work done by F when a particle is taken along the semicircular
path OAB where the coordinates of B are (4, 0) is
y

x
O B
65 75 73
(A) J (B) J (C)
J (D) Zero
3 2 4

118. The potential energy function associated with the force F  4 xyiˆ  2 x 2 ˆj is

(A) U  2 x 2 y (B) U  2 x 2 y (C) U  2 x 2 y  constant (D) Not defined



119. The potential energy for a force field F is given by U  x, y   cos  x  y . The force acting on a
particle at position given by coordinates (0,  / 4) is

1 3 ˆ 1 3 ˆ
(A) 
1
iˆ  ˆj  (B) 
1 ˆ ˆ
ij  (C)  iˆ 
2 2
j  (D)  iˆ 
2 2
j 
2 2    

Raman Niwas, Near Aakashvani, Mahmoorganj, Varanasi. Ph.(0542)–2363455,website www.catjee.in


[51]
120. A particle, which is constrained to move along the x – axis, is subjected to a force in the same direction
which varies with the distance x from the origin as F  x   kx  ax 2 . Here k and a are positive

constants. For x  0 , the functional form of the potential energy U  x  of the particle is:

U(x) U(x)

(A) (B)
x x

U(x) U(x)

(C) (D)
x x


121. The potential energy for a force field F is given by U  x, y   sin  x  y . The magnitude of force


acting on the particle of mass m at  0,  is:
 4
1
(A) 1 (B) 2 (C) (D) 0
2

122. The potential energy in joules of a particle of mass 1kg moving in a plane is given by U  3x  4 y, the
position coordinates of the point being x and y, measured in metres. If the particle is at rest at (6, 4),
then

(A) Its acceleration is of magnitude 5 m/s2


(B) Its velocity when it crosses the y – axis is 10 m/s
(C) It crosses the y-axis (x = 0) at y  4

(D) It moves in a straight line passing through the origin (0, 0)

Raman Niwas, Near Aakashvani, Mahmoorganj, Varanasi. Ph.(0542)–2363455,website www.catjee.in


[52]
ANSWER KEY

87. [ mg  3cos  2cos0  ; 60 ] 200


101. [(i) 125 J; (ii) 50 J; (iii) J;
r 3
88. (iv) non-conservative force.]
2
3mgr
mgR 2 l 
102. [ sin   ]
89. l R
k
90. 5.42 m/s; 0.96 m 103. (C)
104. (A)
2
91. sin 1   105. (A)
3 106. (A)
92. 1.167 m 107. (B)
93. mid point 108. (C)
94. m = 0.5 kg, v  ax3/2 , a  5 m1/2s1/2 ,W  ? 109. (B)
110. (A)
95.   2 yiˆ   2 x  z  ˆj  ykˆ  111. (A)
  112. (B)
2
b 113. (D)
96.
4a 114. (B)
97. 5 ms .
–1 115. (D)
116. (A, C)
6U 0   1/ 6
13 7
 a a  117. (D)
98. F 2       ; 2 a
a   r r  118. (B)
99. [(i) 30.0 J; (ii) –51.2 J ; (iii) –42.4 J; (iv) 119. (B)
The force of friction is a non-conservative 120. (D)
force] 121. (A)
100. [(i) –200 J ; (ii) –200J; (iii) 200 J] 122. (A, B, C)

Exercise1
[Type 1]
1. A small block of mass m is kept on a rough inclined surface of inclination  fixed in a lift. The lift
moves up with a uniform velocity v and the block does not slide on the incline. The work done by the
force of friction on the block in time t will be
(A) Zero (B) mgvt cos2  (C) mgvt sin 2  (D) mgvt sin 2
2. Consider two observers moving with respect to each other at a speed v along a straight line. They
observe a block of mass m moving a distance  on a rough surface. The following quantities will be
same as observed by the observers
(A) Kinetic energy of the block at time t. (B) Work done by friction.
(C) Total work done on the block. (D) Acceleration of the block.
3. The force acting on a body moving along x-axis varies with the position of the particle as shown in
figure. The body is in stable equilibrium at
F

x1 x2 x

(A) x  x1 (B) x  x2
(C) Both x  x1 and x  x2 (D) Neither at x  x1 nor at x  x2

Raman Niwas, Near Aakashvani, Mahmoorganj, Varanasi. Ph.(0542)–2363455,website www.catjee.in


[53]
4. A uniform chain has mass M and length . It is lying on a smooth horizontal table with half of its length
hanging vertically downward. The work done in pulling the chain up the table is
Mg Mg Mg Mg
(A) (B) (C) (D)
2 4 8 16
5. A block is resting over a smooth horizontal plane. A constant horizontal force starts acting on it at t  0
. Which of the following graph is correct
Ek Ek Ek

Power
(A) (B) (C) (D)
0 t 0 t 0 Displacement 0 Displacement

6. If the block in the shown arrangement is acted upon by a constant force F for t  0, its maximum speed
will be
k m
F

F 2F F 2F
(A) (B) (C) (D)
mk mk 2mk mk
7. A block hangs freely from the end of a spring. A boy then slowly pushes the block upwards so that the
string becomes strain free. The gain in gravitational potential energy of the block during the process is
equal to
(A) The work done by the boy against the gravitational force acting on the block
(B) The loss of energy stored in the spring minus the work done by the tension in the spring.
(C) The work done on the block by the boy plus the loss of energy stored in the spring.
(D) The work done on the block by the boy minus the work done by the tension in the spring plus the
loss of energy stored in the spring.
(E) The work done on the block by the boy minus the work done by the tension in the spring.
8. A block of mass m is released from rest at point A. The compression in spring, when the speed of block
is maximum
A
m

k 
smooth

mg sin  2mg sin  mg cos mg


(A) (B) (C) (D)
k k k k
9. A light spring is hung vertically from a fixed support and a heavy mass is attached to its lower end. The
mass is then slowly lowered to its equilibrium position. This stretches the spring by an amount d. If the
same body is permitted to fall instead, through what distance does it stretch the string?
(A) d (B) 1.5d (C) 2d (D) 3d
10. A running man has half the kinetic energy of a boy of half his mass. The man speeds up by 1 m/sec and
then has the same kinetic energy as the boy. The original speed of the boy was
(A) 2 m/s (B) 9.6 m/s (C) 4.8 m/s (D) 7.2 m/s
11. An elastic string of un-stretched length  and force constant k is stretched by a small length x. It is
further stretch by small length y. The work done in the second stretching is
(A) 1 ky 2
2 2 
(B) 1 k x 2  y 2  (C) 1 k  x  y 
2
2
(D) 1 ky  2 x  y 
2
Raman Niwas, Near Aakashvani, Mahmoorganj, Varanasi. Ph.(0542)–2363455,website www.catjee.in
[54]
o
12. A ball P is projected vertically up. Another similar ball Q is projected at an angle 45 . Both reach the
same height during their motion. Then, at the starting point, ratio of kinetic energy of P and Q is?
(A) 0.50 (B) 0.25 (C) 2 (D) 4
13. A pendulum of mass 1 kg and length   1 m is released from rest at angle 60o. The power delivered by
all the forces acting on the bob at an angle   30o is ( g  10 m/s2)
(A) 1.34 Watt (B) 13.4 Watt (C) 0.67 Watt (D) 5 Watt
14. A stone of mass m, tied to the end of a string, is whirled around in a horizontal circle (neglect gravity).
The length of the string is reduced gradually such that mvr  constant. Then, the tension in the string is
given by T  Ar n , where A is a constant and r is the instantaneous radius of the circle. Then, n is equal
to
(A) +2 (B) 2 (C) +3 (D) 3
15. A block of mass M released from rest from point O as shown below. The velocity of the block at the
lowest points are VD ,VE,VF respectively. Assume coefficient of kinetic friction between surface and the
block is same in all cases. Then,
O O O

y y y

x D x E x F
(A) (B) (C)
(A) VD  VE  VF (B) VF  VE  VD

(C) VD  VE  VF  0 (D) VD  VE  VF  0

16. A particle of mass m is moving in a circular path of constant radius r such that its centripetal acceleration ac
is varying with time as ac  k 2 rt 2 , where k is a constant. The power delivered to the particle by the forces
acting on it is

mk 4 r 2t 5
(A) 2 mk 2 r 2t (B) mk 2 r 2t (C) (D) Zero
5
17. A body of mass m is moving in a circle of radius r with a constant speed v. The force on the body is
mv 2
and is directed towards the center. If work done by this force in moving the body over half the
r
circumference and complete circumference is W and W , Then
mv 2 mv 2
(A) W   r , W '  zero (B) W  2r , zero
r r
mv 2 mv 2
(C) W  , W'  .2 r 2 (D) W = zero, W '  Zero
r r
18. A particle of mass m is whirled in a vertical circle with the help of a thread. If the maximum tension in
the thread is double its minimum value then the value of minimum tension in the thread will be
(A) 6mg (B) zero (C) 3mg (D) can‘t be found
19. A block of mass m is pulled by a constant power P placed on a rough horizontal plane. The friction
coefficient between the block and surface is . The maximum velocity of the block is
P P P P
(A) (B) (C) (D) 2
mg mg  mg  mg
Raman Niwas, Near Aakashvani, Mahmoorganj, Varanasi. Ph.(0542)–2363455,website www.catjee.in
[55]
20. A block of mass 100 g moved with a speed of 5 m/s at the highest point in a closed circular tube of
radius 10 cm kept in a vertical plane. The crosssection of the tube is such that the block just fits in it.
The block makes several oscillations inside the tube and finally steps at the lowest point. The work
done by the tube on the block during the process is
(A) 1.45 J (B) 1.45 J (C) 0.2 J (D) zero
21. A heavy stone is thrown from a cliff of height h with a speed v. The stone will hit the ground with
maximum speed if it is thrown
(A) vertically downward (B) vertically upward
(C) horizontally (D) the speed does not depend on the initial direction
22. Two springs A and B  k A  2kB  are stretched by applying forces of equal magnitudes at the four ends.
If the energy stored in A is E, that in B is
(A) E (B) 2E (C) E (D) E
2 4
23. Two equal masses are attached to the two ends of a spring of spring constant k. The masses are pulled
out symmetrically to stretch the spring by a length x over its natural length. The work done by the
spring on each mass is
1 2 1 1 2 1
(A) kx (B)  kx 2 (C) kx (D)  kx 2
2 2 4 4
24. The negative of the work done by the conservative internal forces on a system equals the change in
(A) total energy (B) kinetic energy (C) potential energy (D) none of these
25. The work done by the external forces on a system equals the change in
(A) total energy (B) kinetic energy (C) potential energy (D) none of these

26. The work done by all the forces (external and internal) on a system equals the change in
(A) total energy (B) kinetic energy (C) potential energy (D) none of these
27. _____________of a two particle system depends only on the separation between the two particles. The
most appropriate choice for the blank space in the above sentence is :
(A) kinetic energy (B) total mechanical energy
(C) potential energy (D) total energy
28. A block of mass m slides down a smooth vertical circular track. During the motion, the block is in
(A) vertical equilibrium (B) horizontal equilibrium
(C) radial equilibrium (D) none of these
29. A particle is rotated in a vertical circle by connecting it to a string of length  and keeping the other end

of the string fixed. The minimum speed of the particle when the string is horizontal for which the
particle will complete the circle is

(A) g (B) 2g (C) 3g (D) 5g

Raman Niwas, Near Aakashvani, Mahmoorganj, Varanasi. Ph.(0542)–2363455,website www.catjee.in


[56]
30. In the shown diagram mass of A is m and that of B is 2m. All the surfaces are smooth. System is
released from rest with spring unstretched. Then, the maximum extension ( xm ) in spring will be

k A

mg 2mg 3mg 4mg


(A) (B) (C) (D)
k k k k
xm
31. In above question speed of block A when the extension in spring is
2 is

m 2m 2m 4m
(A) 2 g (B) 2 g (C) 2 g (D) g
k k 3k 3k
32. A chain of length L and mass M is arranged as shown in following four cases. The correct decreasing
order of potential energy (assumed zero at horizontal surface) is

(i) (ii) (iii) (iv) (v)

(A) i  ii  iii  iv  v (B) i  ii  iii  iv  v


(C) i  ii  iv  iii  v (D) i  ii  iv  v  iii
33. A block of mass M is hanging over a smooth and light pulley through a light string. The other end of the
string is pulled by a constant force F. The kinetic energy of the block increases by 20 J in 1s.
(A) The tension in the string is Mg.
(B) The tension in the string is F.
(C) The work done by the tension on the block is 20 J in the above 1 sec.
(D) The work done by the force of gravity is 20 J in the above 1 sec.

34. A block slides down an inclined plane of inclination  with constant velocity. It is then projected up the
plane with an initial speed u. How far up the incline will it move before coming to rest?

u2 u2 u2 u2
(A) (B) (C) (D)
4 g sin  g sin  2 g sin  4g

Raman Niwas, Near Aakashvani, Mahmoorganj, Varanasi. Ph.(0542)–2363455,website www.catjee.in


[57]
35. A smooth narrow tube is in form of an arc AB of a circle of center at O and radius R is fixed so that A is
vertically above O and OB is horizontal. Particles P and Q of mass m and 2m respectively with an ideal
R
string of length , connecting them is placed as shown in the figure. The speed of the particles as P
2
reaches B will be
A
P

B
O R Q

2 gR 2 gR 2(1   ) gR 2 gR
(A) (B) (C) (D)
3 3 3 3

[Type 2]

1. A heavy stone is thrown from a cliff of height h in a given direction. The speed with which it strikes the
ground
(A) Must depend on the speed of projection. (B) Must be larger than the speed of projection.
(C) Must depend upon direction of projection. (D) Must be smaller than the speed of projection.
2. One end of a light spring of spring constant k is fixed to a wall and the other end is tied to a block
placed on a smooth horizontal surface. In a displacement, the work done by the spring is 1 kx 2 . The
2
possible cases are
(A) The spring was initially compressed by a distance x and was finally in its natural length.
(B) It was initially stretched by a distance x and finally was in its natural length.
(C) It was initially in its natural length and finally in a compressed position.
(D) It was initially in its natural length and finally in a stretched position.
3. A vehicle is driven along a straight horizontal track by a motor which exerts a constant driving force.
The vehicle starts from rest at t  0 and the effects of friction and air resistance are negligible. If kinetic
energy of vehicle at time t is E and power delivered by the motor is P, which of the following graphs
is/are correct

P P

(A) (B)

0 t 0 t

Raman Niwas, Near Aakashvani, Mahmoorganj, Varanasi. Ph.(0542)–2363455,website www.catjee.in


[58]
E E

(C) (D)

0 t 0 t

4. Consider two observers moving with respect to each other at a speed v along a straight line. They
observe a block of mass m moving a distance  on a rough surface. The following quantities will be
same as observed by the two observers
(A) kinetic energy of the block at time t (B) work done by friction
(C) total work done on the block (D) acceleration of the block
5. One end of a light spring of spring constant k is fixed to a wall and the other end is tied to a block
1
placed on a smooth horizontal surface. In a displacement, the work done by the spring is kx 2 . The
2
possible cases are
(A) the spring was initially compressed by a distance x and was finally in its natural length
(B) it was initially stretched by a distance x and finally was in its natural length.
(C) it was initially in its natural length and finally in a compressed position
(D) It was initially in its natural length and finally in a stretched position.
6. Which of the following is/are correct
(A) Work done by static friction is always zero
(B) Work done kinetic friction can be positive also
(C) Kinetic energy of a system can not be increased without applying any external force on the system.
(D) Work energy theorem is valid in non-inertial frame also.
7. The kinetic energy of a particle continuously increases with time
(A) The resultant force on the particle must be parallel to the velocity at all the instants.
(B) The resultant force on the particle must be at an angle less than 90o all the times.
(C) Its height above the ground level must continuously decreases.
(D) The magnitude of its linear momentum is increasing continuously.
8. You lift a suitcase from the floor and keep it on a table. The work done by you on the suitcase does not
depend on
(A) the path taken by the suitcase (B) the time taken by you in doing so
(C) the weight of the suitcase (D) your weight
9. A particle of mass m is attached to a light string of length , the other end of which is fixed. Initially the
string is kept horizontal and the particle is given an upward velocity v. The particle is just able to
complete a circle.
(A) The string becomes slack when the particle reaches its highest point.
(B) The velocity of the particles becomes zero at the highest point.

(C) The kinetic energy of the ball in initial position was 1 mv 2  mg  .


2
(D) The particle again passes through the initial position.

Raman Niwas, Near Aakashvani, Mahmoorganj, Varanasi. Ph.(0542)–2363455,website www.catjee.in


[59]
10. Spring A and B are identical except that A is stiffer than B. If work expended in spring A and B are WA
and WB, when they are stretched by same amount, while work expend in spring A and B are W ’A and
W ’B when they are stretched by the same force then

(A) WA  WB (B) WA  WB (C) W ' A  W ' B (D) W ' A  W ' B

11. The potential energy U in joule of a particle of mass 1 kg moving in x-y plane obeys the law
U  3x  4 y , where (x, y) are the co-ordinates of the particles in meter. If the particle is at rest at (6, 4)

at time t  0 then

(A) the particle has constant acceleration

(B) the particle has zero acceleration

(C) the speed of the particle when it crosses y-axis is 10 m/s

(D) co-ordinate of particle at t  1 sec is (4.5, 2)

12. A particle of mass m is attached to a light string of length , the other end of which is fixed. Initially the

string is kept horizontal and the particle is given an upward velocity v. The particle is just able to
complete a circle.

(A) The string becomes slack when the particle reached its highest point.

(B) The velocity of the particle becomes zero at the highest point.

1 2
(C) The kinetic energy of the ball in initial position was mv  mg  .
2

(D) The particle again passes through the initial position



13. An object is displaced from a point A (0, 0, 0) to B (1 m, 1 m, 1 m) under a force F  ( y ˆi  x ˆj) . The
work done by this force and the nature of the force is

(A) 1 J, Non-conservative (B) 1 J, Conservative

(C) zero , Conservative (D) zero , Non-conservative

14. A particle mass is tied to an ideal string and whirled in a vertical circle of radius R, where R is the
length of the string. If the ratio of the maximum to minimum tension in the string throughout the motion
is 2 : 1, then the maximum possible speed of the particle will be

(A) 11gR (B) 5gR (C) 10gR (D) 3gR

Raman Niwas, Near Aakashvani, Mahmoorganj, Varanasi. Ph.(0542)–2363455,website www.catjee.in


[60]
15. The following plot shows the variation of potential energy (U) of a system versus position (x). From the
graph we can interpret that
U

0 C
x
A B D

(A) Point D is position of neutral equilibrium (B) Point B is position of unstable equilibrium
(C) Point C is position of stable equilibrium (D) Point A is position of neutral equilibrium
16. In a children‘s park, there is a slide which has a total length of 10 m and a height of 8 m. A vertical
ladder is provided to reach the top. A boy weighing 200 N climbs up the ladder to the top of the slide
and slides down to the ground. The average friction offered by the slide is three tenth of his weight.
Then
(A) The work done by ladder on the boy as he goes up is zero.
(B) The work done by ladder on boy as he goes up is 1600 J
(C) The work done by slide on boy as he comes down is 600 J
(D) The work done by slide on boy as he comes down is 1600 J
17. A particle of mass m is kept at the top of a smooth fixed sphere. It is given a horizontal velocity v then
(A) it will start moving along a circular path if v  gR

(B) it will start moving along a circular path if v  gR

(C) it will start moving along a parabolic path if v  gR

(D) it will start moving along a parabolic path if v  gR


18. The total work done on a particle is equal to the change in its kinetic energy
(A) always (B) only if the forces acting on it are conservative
(C) only if gravitational force alone acts on it (D) only if elastic force along acts on it.
19. A particle is acted upon by a force of constant magnitude which is always perpendicular to the velocity
of the particle. The motion of the particle takes place in a plane. If follows that
(A) its velocity is constant (B) its acceleration is constant
(C) its kinetic energy is constant (D) it moves in a circular path
20. A self-propelled vehicle of mass m, whose engine delivers a constant power P, has an acceleration
P
a . (Assume that there is no friction). In order to increase its velocity from v1 to v2 , the distance it
mv
has to travel will be
3P 2 m 3 m 2 m
(A) (v2  v12 ) (B) (v2  v13 ) (C) (v2  v12 ) (D) (v2  v1 )3
m 3P 3P 3P
21. A ball of mass 50 g and relative density 0.5 strikes the surface of the water with a velocity of 20 m/sec.
It comes to rest at a depth of 2 m. Find the work done by the resisting force in water: (take g  10 m/s2)
(A) 6 J (B) +7.5 J (C) 9 J (D) 10 J

Raman Niwas, Near Aakashvani, Mahmoorganj, Varanasi. Ph.(0542)–2363455,website www.catjee.in


[61]
22. A body is moved along a straight line by a machine delivering constant power. The distance moved by
the body in time t is proportional to
1 3 3
(A) t 2 (B) t 4 (C) t 2 (D) t2
23. A body with mass 2 kg moves in one direction in the presence of a force which is described by the
potential energy graph. If the body is released from rest at x  2 m, then its speed when it crosses
x  5 m is
10
8

U(Jouse)
6
4
2
x(meter)
1 2 3 4 5
(A) zero (B) 1 ms–1 (C) 2 ms–1 (D) 3 ms–1
24. Three blocks A, B and C are kept as shown in the figure. The coefficient of friction between A and B
is 0.2, B and C is 0.1, C and ground is 0.0. The mass of A, B and C are 3 kg, 2 kg and 1 kg respectively.
A is given a horizontal velocity 10 m/s. A, B and C always remain in contact i.e. lies as in figure. The
total work done by friction will be
A
B
C smooth

(A) –75 J (B) 75 J (C) –150 J (D) –100 J


25. 1 kg block collides with a horizontal mass-less spring of force constant 2 N/m. The block compresses
the spring by 4 m. If the coefficient of kinetic friction between the block and the surface is 0.25, what
was the speed of the block at the instant of collision? (take g  10 m/s2)

k m

(A) 7.2 m/s (B) 4 2 m/s (C) 4.5 m/s (D) 10 m/s
26. A stone with weight W is thrown vertically upward into the air with initial velocity v0 . If a constant
force  due to air drag acts on the stone throughout the flight and if the maximum height attain by stone
is h and velocity when it strikes to the ground is v. Which one is correct?

(A) h  v0 2
1  f W  , v  v (B) h 
v0 2
, v  zero
 
0
2g 2g 1  f
W

v0 2 Wf v0 2 Wf
(C) h  , v  v0 (D) h  , v  v0

2g 1  f
W  Wf

2g 1  f
W  Wf

Raman Niwas, Near Aakashvani, Mahmoorganj, Varanasi. Ph.(0542)–2363455,website www.catjee.in


[62]
27. In the shown figure, a small mass m starts sliding down a smooth and stationary circular track. Which
of the following graph best represents the variation of magnitude of the force applied by the track on the
mass and the angle ?

m

F F
F F
3mg  2mg 
3mg  mg 
(A) (B) (C) (D)
0    
0  0  0   
2       
2 2 2

28. In the Q No. 27, if M  2m and friction exists between the circular track and the horizontal surface
then, which of the following plot best represents the variation of frictional force versus the angle 
F F

3mg 
mg
(A) (B) 
 2 
0   0  
2 2

F F

(C)
3 2 mg  (D)
3 2 mg  
2
 
0   0 
2

29. Two cylindrical vessels of equal crosssectional area A contain water up-to heights h1 and h2. The vessels
are interconnected so that the levels in them become equal. The work done by the force of gravity during
the process is

h h  h h   Ah1h2
2 2

(A) zero (B)  A  1 2  g (C)  A  1 2  g (D) g


 2   2  2

30. The mass m slides down the track and completes the vertical circle on the smooth curved surface. The
minimum value of h will be

R
h

(A) R (B) 2R (C) 2.5 R (D) 3 R

Raman Niwas, Near Aakashvani, Mahmoorganj, Varanasi. Ph.(0542)–2363455,website www.catjee.in


[63]
31. Two bars of masses m1 and m2 connected by a non-deformed light spring rest on a horizontal plane.
The coefficient of friction between the bars and the surface is equal to . If P is the minimum constant
force that has to be applied in the horizontal direction to the bar of mass m1 in order to shift the other
bar. Then


(A) P   g m1 
m2
2  (B) P   g m2   m1
2 
g
(C) P   g  m1  m2  (D) P   m1  m2 
2
32. A body of mass m was slowly hauled up the rough hill by a force F which at each point was directed
along tangent to the hill. Work done by the force

(A) Is independent of shape of trajectory.


(B) Depends upon vertical component of displacement but independent of horizontal component.
(C) Depends upon both the components of displacement
(D) Does not depend upon coefficient of friction.
33. A chain of mass m and length  rests on a rough surfaced table so that one of its ends hangs over the
edge. The chain starts sliding off the table all by itself provided the overhanging part equals     
part of the chain length. If the total work performed by the friction forces acting on the chain by the
moment it slides completely off the table is W and the friction coefficient between table and chain is .
Then
   (1   )2 mg  (1   )mg 
(A)   (B)   (C) W  (D) W 
1 1  2 2
34. A smooth track in form of a quarter circle of radius 6m lies in the vertical plane. A particle moves from
   
P1 to P2 under the action of forces F1 , F2 and F3 . Force F1 is always towards P2 and is always 20 N in
 
magnitude. Force F2 always acts horizontally and is always 30 N in magnitude. Force F3 always acts
tangentially to the track and is of magnitude 15 N. Select the correct magnitude:
6m
O P2

6m F1 
F3

F2
P1
 
(A) Work done by F1 is 120 J (B) Work done by F2 is 180 J
 
(C) Work done by F3 is 45 J (D) F1 is conservative in nature

Raman Niwas, Near Aakashvani, Mahmoorganj, Varanasi. Ph.(0542)–2363455,website www.catjee.in


[64]
Exercise  2 [JEE Main/AIEEE Asked]

1. A block of mass 0.18 kg is attached to a spring of force-constant 2 N/m. The coefficient of friction
between the block and the floor is 0.1. Initially the block is at rest and the spring is un-stretched. An
impulse is given to the block as shown in the figure. The block slides a distance of 0.06 m and comes to
n
rest for the first time. The initial velocity of the block in m/s is v  . Then n is
10
m k

2. A spring of force constant 800 N/m has an extension of 5 cm. The work done in extending it from 5 to
15cm is
[AIEEE - 02]
(A) 16 J (B) 8 J (C) 32 J (D) 24 J
3. A ball whose kinetic energy is E is projected at an angle 45o with horizontal. The kinetic energy of the
ball at highest point of its flight is
[AIEEE - 02]

(A) E (B) E (C) E 2 (D) zero


2

4. From a building two balls A and B are thrown such that A is thrown upwards and B downwards (both
with same speed). If V A and VB are their respective velocities on reaching the ground, then
[AIEEE - 02]
(A) VB  VA (B) VB  VA
(C) VB  VA (D) their velocities depends upon their masses
5. Consider the following two statements : [AIEEE - 03]
a. Linear momentum of a system of particles is zero
b. Kinetic energy of a system of particles is zero
Then
(A) A does not imply b and b does not imply a
(B) A implies b but b does not imply a
(C) a does not imply b but b implies a
(D) a implies b and b implies a
6. A wire suspended vertically from one of ends is stretched by attaching a weight of 200 N to the lower
end. The weight stretches the wire by 1 mm. Then the elastic energy stored in the wire is
[AIEE-2003]
(A) 0.2 J (B) 10 J (C) 20 J (D) 0.1 J
7. A spring of spring constant 5000 N/m is stretched initially by 5 cm from the un-stretched position. The
work required to stretch further by another 5 cm is
[AIEEE - 03]
(A) 12.5 J (B) 18.75 J (C) 25 J (D) 6.25 J
8. A body is moved along a straight line by a machine delivering constant power. The distance moved by
the body in time t is proportional to
[AIEEE - 03]
3 3 1 1
(A) t 4 (B) t 2 (C) t 4 (D) t 2
9. A particle moves in a straight line with retardation proportional to its displacement. Its loss of kinetic
energy for any displacement x is proportional to
[AIEEE - 04]
x 2
(A) x (B) e (C) x (D) ln x
Raman Niwas, Near Aakashvani, Mahmoorganj, Varanasi. Ph.(0542)–2363455,website www.catjee.in
[65]
10. A particle is acted upon by a force of constant magnitude which is always perpendicular to the velocity
of the particle, the motion of the particles takes place in a plane. It follows that [AIEEE -04]
(A) its kinetic energy is constant (B) its acceleration is constant
(C) its velocity is constant (D) it moves in a straight line
11. A uniform chain of length 2 m is kept on table such that a length of 60 cm hangs freely from the edge of
the table. The total mass of the chain is 4 kg. What is the work done in pulling the entire chain on the
table ( g  10 m/s2) [AIEEE - 04]

(A) 12 J (B) 3.6 J (C) 7.2 J (D)1200 J



12. A force F  (5 ˆi  3 ˆj  2 kˆ ) N is applied over a particle which displaces it from the origin to the point

r  (2 ˆi  ˆj) m. The work on the particle in joules is [AIEEE - 04]

(A) +10 (B) +7 (C) 7 (D) 13


13. A body of mass m accelerates uniformly from rest to v1 in time t1 . The instantaneous power delivered
to the body as a function of time is [AIEEE - 04]
mv1t 2 mv12t mv1t mv12t
(A) (B) (C) (D)
t1 t12 t1 t1

14. A wire fixed at upper end stretches by a length  by applying a force F. The work done in stretching is
[AIEEE - 04]
F F
(A) 2F  (B) F  (C) (D)
2 2
15. The block of mass m moving on the frictionless horizontal surface collides with the spring of spring
constant k and compresses it by a length . The maximum momentum of the block after collision is
[AIEEE - 05]
k m

k 2 m 2
(A) mk  (B) (C) zero (D)
2m k
16. A body of mass m accelerates uniformly from rest to v in time T. The instantaneous power delivered to
the body as a function of time is
[AIEEE - 05]
2 2
mv 2
mv 2
1 mv 1 mv
(A) t (B) t (C) t (D) t
T T2 2 T 2 T2
17. A mass of M kg is suspended by a weightless string. The horizontal force that is required to displace it
until the string makes an angle of 45o with the initial vertical direction is
[AIEEE - 06]
Mg
(A) Mg  2  1 (B) Mg 2 (C) (D) Mg  2  1
2
18. A particle of mass 100 g is thrown vertically upwards with a speed of 5 m/s. The work done by the
force of gravity during the time the particle goes up is
[AIEEE - 06]
(A) 0.5 J (B) 1.25 J (C) 1.25 J (D) 0.5 J

Raman Niwas, Near Aakashvani, Mahmoorganj, Varanasi. Ph.(0542)–2363455,website www.catjee.in


[66]
19. A player caught a cricket ball of mass 150 g moving at a rate of 20 m/s. If the catching process is
completed in 0.1 s, the force of the blow exerted by the ball on the hand of the player is equal to
[AIEEE - 06]
(A) 150 N (B) 3 N (C) 30 N (D) 300 N
 x4 x2 
20. The potential energy of a 1 kg particle free to move along the xaxis is given by V  x      J
 4 2 
The total mechanical energy of the particle is 2 J. Then, the maximum speed (in m/s) is
[AIEEE - 06]
(A) 3 (B) 2 (C) 1 (D) 2
2 2
21. A 2 kg block slides on a horizontal floor with a speed of 4 m/s. It strikes a uncompressed spring, and
compresses it till the block is motionless. The kinetic friction force is 15N and spring constant is
10,000 N/m. The spring compresses by
[AIEEE - 07]
(A) 5.5 cm (B) 2.5 cm (C) 11.0 cm (D) 8.5 cm
o
22. A particle is projected at 60 to the horizontal with a kinetic energy K. The kinetic energy at the highest point
is
[AIEEE - 07]
(A) K (B) zero (C) K 2 (D) K 4

23. An athlete in the Olympic games covers distance of 100 m in 10 s. His kinetic energy can be estimated
to be in a range [AIEEE - 08]
(A) 200 J  500 J (B) 2  105 J  3  105 J
(C) 20,000 J  50,000 J (D) 2,000 J  5,000 J
24. The potential energy function for the force between two atoms in a diatomic molecule is approximately
a b
given by atoms in a diatomic molecule is approximately given by U(x) = 12
 6 , where a and b are
x x
constants and x is the distance between atoms. If the dissociation energy of the molecule is
D  [U ( x  )  U at equilibrium ], D is [AIEEE-2010]
b2 b2 b2 b2
(A) (B) (C) (D)
2a 12a 4a 6a
25. This question has statement 1 and statement 2. Of the four choices given after the Statements, choose
the one that best describes the two Statements.
If two springs S1 and S 2 of force constant k1 and k2 , respectively, are stretched by the same force, it is
found that more work is done on spring S1 than on spring S 2 .
Statement 1 : If stretched by the same amount work done on S1 , work done on S1 is more than S 2
Statement 2 : k1  k2 [2012]
(A) Statement 1 is false, Statement 2 is true
(B) Statement 1 is true, Statement 2 is false
(C) Statement 1 is true, Statement 2 is true, Statement 2 is the correct explanation for Statement 1
(D) Statement 1 is true, Statement 2 is true, Statement 2 is not the correct explanation for Statement 1
26. When a rubberband is stretched by a distance x, it exerts restoring force of magnitude F  ax  bx2
where a and b are constants. The work done in stretching the unstretched rubber-band by L is :
[JEE Mains 2014]
1  aL2 bL3 
 aL2  bL3 
2 3
1 aL bL
(A) aL2  bL3 (B) (C)  (D)   
2 2 3 2 2 3 
Raman Niwas, Near Aakashvani, Mahmoorganj, Varanasi. Ph.(0542)–2363455,website www.catjee.in
[67]
27. Two springs of force constants 300 N/m (Spring A) and 400 N/m (Spring B) are joined together in
E
series. The combination is compressed by 8.75 cm. The ratio of energy stored in A and B is A . Then
EB
EA
is equal to : [JEE Mains 2013]
EB
4 16 3 9
(A) (B) (C) (D)
3 9 4 16
th
1
28. A bullet looses   of its velocity passing through one plank. The number of such planks that are
n
required to stop the bullet can be : [JEE Mains 2014]
2 2
n 2n
(A) (B) (C) infinite (D) n
2n  1 n 1
29. A spring of unstretched length 1 has a mass m with one end fixed to a rigid support. Assuming spring to
be made of a uniform wire, the kinetic energy possessed by it if its free end is pulled with uniform
velocity v is : [JEE Mains 2014]
1 1 1
(A) mv 2 (B) mv 2 (C) mv 2 (D) mv 2
2 3 6
30. A particle is moving in a circle of radius r under the action of a force F   r which is directed towards
2

centre of the circle. Total mechanical energy (kinetic energy + potential energy) of the particle is (take
potential energy = 0 for r = 0) : [JEE Mains 2015]
1 5 4
(a)  r 3 (B)  r 3 (C)  r 3 (D)  r 3
2 5 3
31. Velocity-time graph for a body of mass 10 kg is shown in figure. Work-done on the body in first two
seconds of the motion is : [JEE Mains 2016]
v(m/s)
50ms1

(0,0) 10s t(s)


(A) 9300 J (B) 1200 J (C) 4500 J (D) 12000 J
32. A point particle of mass m, moves along the uniformly rough track PQR as shown in the figure. The
coefficient of friction, between the particle and the rough track equals  . The particles is released, from
rest from the point P and it comes to rest at a point R . The energies, lost by the ball, over the parts, PQ
and QR, of the track, are equal to each other, and no energy is lost when particle changes direction from
PQ to QR. The value of the coefficient of friction  and the distance x (= QR), are respectively close to
: [JEE Mains 2016]
P

h=2m

30 R
Horizontal  Q
Surface
(A) 0.29 and 3.5 m (B) 0.29 and 6.5 m (C) 0.2 and 6.5 m (D) 0.2 and 3.5 m

Raman Niwas, Near Aakashvani, Mahmoorganj, Varanasi. Ph.(0542)–2363455,website www.catjee.in


[68]
33. A particle of mass M is moving in a circle of fixed radius R in such a way that its centripetal
acceleration at time t is given by n2 Rt 2 where n is a constant. The power delivered to the particle by the
force acting on it, is [JEE Mains 2016]

1
(A) Mn 2 R 2t 2 (B) M n2 R 2t (C) M nR 2t 2 (D) M nR 2t
2
34. A car of weight W is on an inclined road that rises by 100 m over a distance of 1 Km and applies a
W
constant frictional force on the car. While moving uphill on the road at a speed of 10 ms 1 , the car
20
P
needs power P. If it needs power while moving downhill at speed v then value of v is :
2
[JEE Mains 2016]
(A) 20 ms 1 (B) 5 ms 1 (C) 15 ms 1 (D) 10 ms 1
35. A time dependent force F = 6t acts on a particle of mass 1 kg. If the particle starts from rest, the work
done by the force during the first 1 second will be [JEE Mains 2017]
(A) 9 J (B) 18 J (C) 4.5 J (D) 22 J
36. A body of mass m = 102 kg is moving in a medium and experiences a frictional force F  kv2 . Its
1 2
initial speed is v0  10 ms 1. If, after 10 s, its energy is mv0 , the value of k will be :
8
[JEE Mains 2017]
4 1 1 1 4 1
(A) 10 kg m (B) 10 kg m s 1
(C) 10 kg m (D) 10 kg m1
3

Raman Niwas, Near Aakashvani, Mahmoorganj, Varanasi. Ph.(0542)–2363455,website www.catjee.in


[69]

IIT  JEE/ JEE Advance Asked


1. Two masses of 1 gm and 4 gm are moving with equal kinetic energies. The ratio of the magnitudes of
their linear momenta is [IIT-JEE 1980]
(A) 4 : 1 (B) 2 :1 (C) 1 : 2 (D) 1 : 16
2. Two blocks A and B are connected to each other by a string and a spring ; the string passes over a
frictionless pulley as shown in the figure. Block B slides over the horizontal top surface of a stationary
block C and the block A slides along the vertical side of C, both with the same uniform speed.
B

C
A
The coefficient of friction between the surfaces of blocks is 0.2. Force constant of the spring is 1960
newtons/m. If mass of block A is 2 kg., calculate the mass of block B and the energy stored in the
spring. [IIT-JEE 1982]
3. A body is moved along a straight line of a machine delivering constant power. The distance moved by
the body in time t is proportional to [IIT-JEE 1984]
(A) t1/ 2 (B) t 3/ 4 (C) t 3/ 2 (D) t 2
4. A uniform chain of length L and mass M is lying on a smooth table and one third of its length is
hanging vertically down over the edge of the table. If g is acceleration due to gravity, the work required
to pull the hanging part on to the table is [IIT-JEE 1985]
(A) MgL (B) MgL / 3 (C) MgL / 9 (D) MgL /18
5. A string, with one end fixed on a rigid wall, passing over a fixed frictionless pulley at a distance of 2m
from the wall, has a point mass M  2kg attached to it at a distance of 1m from the wall. A mass
m  0.5 kg attached at the free end is held at rest so that the string is horizontal between the wall and
the pulley and vertical beyond the pulley. What will be the speed with which the mass M will hit the
wall when the mass m is released? [IIT-JEE 1985]
M

m
6. A wind-powered generator converts wind energy into electrical energy. Assume that the generator
converts a fixed fraction of the wind energy intercepted by its blades into electrical energy. For wind
speed v, the electrical power output will be proportional to [IIT-JEE 2000]
2 3 4
(A) v (B) v (C) v (D) v
7. When a rubberband is stretched by a distance x, it exerts restoring force of magnitude F  ax  bx2
where a and b are constants. The work done in stretching the unstretched rubber-band by L is :
[IIT-JEE 2004]
aL2 bL3 1  aL bL3 
2

(B)  aL2  bL3 


1
(A) aL  bL
2 3
(C)  (D)   
2 2 3 2 2 3 
8. Statement - 1 : A block of mass m starts moving on a rough horizontal surface with a velocity v. It
stops due to friction between the block and the surface after moving through a certain distance. The
surface is now tilted to an angle of 30 with the horizontal and the same block is made to go up on the
surface with the same initial velocity v. The decrease in the mechanical energy in the second situation
is smaller than that in the first situation. [IIT-JEE 2007]

Raman Niwas, Near Aakashvani, Mahmoorganj, Varanasi. Ph.(0542)–2363455,website www.catjee.in


[70]
Statement -2: The coefficient of friction between the block and the surface decreases with the increase
in the angle of inclination.
(A) Statement-1 is True, Statement-2 is True: Statement-2 is True; statement - 2 is a correct explanation
for Statement-1
(B) Statement-1 is True, Statement-2 is True: Statement-2 is NOT a correct explanation for
Statement-1
(C) Statement -1 is True, Statement - 2 is False
(D) Statement -1 is False, Statement - 2 is True
9. Two springs of force constants 300 N/m (Spring A) and 400 N/m (Spring B) are joined together in
E
series. The combination is compressed by 8.75 cm. The ratio of energy stored in A and B is A . Then
EB
EA
is equal to :
EB
[IIT-JEE 2013]
4 16 3 9
(A) (B) (C) (D)
3 9 4 16
10. The work done on a particle of mass m by a force,
 
 x y
K ˆ
i j
ˆ
  x 2  y 2 3/ 2 x  y  
2 2 3/ 2 

(K being a constant of appropriate dimensions), when the particle is taken from the point (a, 0) to the
point (0, a) along a circular path of radius a about the origin in the x  y plane is
[JEE Advanced 2013]
2K  K K
(A) (B) (C) (D) 0
a a 2a
Passage (Q. 11  Q. 12) :-
A small block of mass 1 kg is released from rest at the top of a rough track. The track is a circular arc of radius
40 m. The block slides along the track without toppling and a frictional force acts on it in the direction opposite
to the instantaneous velocity. The work done in overcoming the friction up to the point Q, as shown in the
figure below, is 150 J. (Take the acceleration due to gravity, g = 10 ms 2 ) [JEE Advanced 2013]
y
R
P
30

R
Q

x
O
11. The magnitude of the normal reaction that acts on the block at the point Q is
(A) 7.5 N (B) 8.6 N (C) 11.5 N (D) 22.5 N
12. The speed of the block when it reaches the point Q is

(A) 5ms 1 (B) 10 ms 1 (C) 10 3 ms 1 (D) 20 ms 1

Raman Niwas, Near Aakashvani, Mahmoorganj, Varanasi. Ph.(0542)–2363455,website www.catjee.in


[71]
th
1
13. A bullet looses   of its velocity passing through one plank. The number of such planks that are
n
required to stop the bullet can be : [JEE Advanced, 2014]
n2 2n 2
(A) (B) (C) infinite (D) n
2n  1 n 1
14. A spring of unstretched length 1 has a mass m with one end fixed to a rigid support. Assuming spring to
be made of a uniform wire, the kinetic energy possessed by it if its free end is pulled with uniform
velocity v is : [JEE Advanced, 2014]
1 2 1 1
(A) mv (B) mv 2 (C) mv 2 (D) mv 2
2 3 6
15. A tennis ball is dropped on a horizontal smooth surface. It bounces back to its original position after
hitting the surface. The force on the ball during the collision is proportional to the length of
compression of the ball. Which one of the following sketches describes the variation of its kinetic
energy K with time t most appropriately? The figure are only illustrative and not to the scale.
[JEE Advanced 2014]
K K

(A) (B)

t t
K K

(C) (D)

t t
16. A particle of mass 0.2 kg is moving in one dimension under a force that delivers a constant power 0.5
W to the particle. If the initial speed (in ms 1 ) of the particle is zero, the speed (in ms 1 ) after 5 s is
_____.
[JEE Advanced 2014]
17. Consider an elliptical shaped rail PQ in the vertical plane with OP  3 m and OQ  4 m. A block of
mass 1 kg is pulled along the rail from P to Q with a force of 18 N, which is always parallel to line
PQ (see the figure given). Assuming no frictional losses, the kinetic energy of the block when it reaches
Q is (n 10 ) joules. The value of n is ___________.(take acceleration due to gravity = 10 ms 2 )
[JEE Advanced 2014]
Q

4m

90
O 3m P
18. A particle is moving in a circle of radius r under the action of a force F   r 2 which is directed towards
centre of the circle. Total mechanical energy (kinetic energy + potential energy) of the particle is (take
potential energy = 0 for r = 0) : [JEE Advanced 2015]
1 3 5 3 4 3
(a) r (B) r (C) r (D)  r 3
2 5 3
Raman Niwas, Near Aakashvani, Mahmoorganj, Varanasi. Ph.(0542)–2363455,website www.catjee.in
[72]
19. A particle of unit mass is moving along the x-axis under the influence of a force and its total energy is
conserved. Four possible forms of the potential energy of the particle are given in column I (a and U 0
constants). Match the potential energies in column I to the corresponding statement(s) in column II.
Column I Column II
(A) 2 2 (p) The force acting on the particles is zero at x  a
U   x 
U1 ( x)  0 1    
2   a  
(B) U x
2 (q) The force acting on the particle is zero at x = 0
U 2 ( x)  0  
2 a
(C) U  x
2
  x 2  (r) The force acting on the particle is zero at x  a
U 3 ( x)  0   exp     
2 a   a  
(D) U x 1 x   (s) The particle experiences an attractive force towards
3

U 4 ( x)  0      x  0 in the region |x| < a


2  a 3  a  
(t) U
The particle with total energy 0 can oscillate about
4
the point x  a
20. Velocity-time graph for a body of mass 10 kg is shown in figure. Work-done on the body in first two
seconds of the motion is : [JEE Advanced, 2016]
v(m/s)
50ms1

(0,0) 10s t(s)


(A) 9300 J (B) 1200 J (C) 4500 J (D) 12000 J
21. A point particle of mass m, moves along the uniformly rough track PQR as shown in the figure. The
coefficient of friction, between the particle and the rough track equals  . The particles is released, from
rest from the point P and it comes to rest at a point R . The energies, lost by the ball, over the parts, PQ
and QR, of the track, are equal to each other, and no energy is lost when particle changes direction from
PQ to QR. The value of the coefficient of friction  and the distance x (= QR), are respectively close to
[JEE Advanced 2016]
P

h=2m

30 R
Horizontal  Q
Surface
(A) 0.29 and 3.5 m (B) 0.29 and 6.5 m (C) 0.2 and 6.5 m (D) 0.2 and 3.5 m
22. A person trying to lose weight by burning fat lifts a mass of 10 kg upto a height of 1 m 1000 times.
Assume that the potential energy lost each time he lowers the mass is dissipated. How much fat will he
use up considering the work done only when the weight is lifted up? Fat supplies 3.8 107 J of energy
per kg which is converted to mechanical energy with a 20% efficiency rate. Take g  9.8ms2 :
[JEE Advanced 2016]

(A) 9.89 103 kg (B) 12.89 103 kg (C) 2.45 103 kg (D) 6.45 103 kg

Raman Niwas, Near Aakashvani, Mahmoorganj, Varanasi. Ph.(0542)–2363455,website www.catjee.in


[73]
23. A particle of mass M is moving in a circle of fixed radius R in such a way that its centripetal
acceleration at time t is given by n2 Rt 2 where n is a constant. The power delivered to the particle by the
force acting on it, is [JEE Advanced 2016]
1
(A) Mn 2 R 2t 2 (B) M n2 R 2t (C) M nR 2t 2 (D) M nR 2t
2
24. A car of weight W is on an inclined road that rises by 100 m over a distance of 1 Km and applies a
W
constant frictional force on the car. While moving uphill on the road at a speed of 10 ms 1 , the car
20
P
needs power P. If it needs power while moving downhill at speed v then value of v is :
2
[JEE Advanced 2016]
(A) 20 ms 1 (B) 5 ms 1 (C) 15 ms 1 (D) 10 ms 1
25. A time dependent force F = 6t acts on a particle of mass 1 kg. If the particle starts from rest, the work
done by the force during the first 1 second will be [JEE Advanced 2017]
(A) 9 J (B) 18 J (C) 4.5 J (D) 22 J
2
26. A body of mass m = 10 kg is moving in a medium and experiences a frictional force F  kv2 . Its
1
initial speed is v0  10 ms 1. If, after 10 s, its energy is mv02 , the value of k will be :
8
[JEE Advanced 2017]
(A) 104 kg m1 (B) 101 kg m1 s 1 (C) 104 kg m1 (D) 103 kg m1

Exercise  3

1. A self-propelled vehicle of mass m, whose engine delivers a constant power P, has an acceleration
P
a . (Assume that there is no friction). In order to increase its velocity from v1 to v2 , the distance it
mv
has to travel will be
3P 2 m 3 m 2 m
(A) (v2  v12 ) (B) (v2  v13 ) (C) (v2  v12 ) (D) (v2  v1 )3
m 3P 3P 3P
2. A ball of mass 50 g and relative density 0.5 strikes the surface of the water with a velocity of 20 m/sec.
It comes to rest at a depth of 2 m. Find the work done by the resisting force in water: (take g  10 m/s2)
(A) 6 J (B) +7.5 J (C) 9 J (D) 10 J
3. A body is moved along a straight line by a machine delivering constant power. The distance moved by
the body in time t is proportional to
1 3 3
(A) t 2 (B) t 4 (C) t 2 (D) t2
4. A body with mass 2 kg moves in one direction in the presence of a force which is described by the
potential energy graph. If the body is released from rest at x  2 m, then its speed when it crosses
x  5 m is
10
8
U(Jouse)

6
4
2
x(meter)
1 2 3 4 5
(A) zero (B) 1 ms–1 (C) 2 ms–1 (D) 3 ms–1
Raman Niwas, Near Aakashvani, Mahmoorganj, Varanasi. Ph.(0542)–2363455,website www.catjee.in
[74]
5. Three blocks A, B and C are kept as shown in the figure. The coefficient of friction between A and B
is 0.2, B and C is 0.1, C and ground is 0.0. The mass of A, B and C are 3 kg, 2 kg and 1 kg respectively.
A is given a horizontal velocity 10 m/s. A, B and C always remain in contact i.e. lies as in figure. The
total work done by friction will be
A
B
C smooth

(A) –75 J (B) 75 J (C) –150 J (D) –100 J


6. 1 kg block collides with a horizontal mass-less spring of force constant 2 N/m. The block compresses
the spring by 4 m. If the coefficient of kinetic friction between the block and the surface is 0.25, what
was the speed of the block at the instant of collision? (take g  10 m/s2)
k m

(A) 7.2 m/s (B) 4 2 m/s (C) 4.5 m/s (D) 10 m/s

7. A stone with weight W is thrown vertically upward into the air with initial velocity v0 . If a constant

force  due to air drag acts on the stone throughout the flight and if the maximum height attain by stone
is h and velocity when it strikes to the ground is v. Which one is correct?

(A) h  v0 2
1  f W  , v  v (B) h 
v0 2
, v  zero
 
0
2g 2g 1  f
W

v0 2 Wf v0 2 Wf
(C) h  , v  v0 (D) h  , v  v0

2g 1  f
W  Wf

2g 1  f
W  Wf

8. In the shown figure, a small mass m starts sliding down a smooth and stationary circular track. Which
of the following graph best represents the variation of magnitude of the force applied by the track on the
mass and the angle ?


m

Raman Niwas, Near Aakashvani, Mahmoorganj, Varanasi. Ph.(0542)–2363455,website www.catjee.in


[75]
9. In the Q No. 10, if M  2m and friction exists between the circular track and the horizontal surface
then, which of the following plot best represents the variation of frictional force versus the angle 

F F

3mg 
mg
(A) (B) 
 2 
0   0  
2 2

F F

(C)
3 2 mg  (D)
3 2 mg  
2
 

0   0
2

10. Two cylindrical vessels of equal crosssectional area A contain water up-to heights h1 and h2. The vessels
are interconnected so that the levels in them become equal. The work done by the force of gravity during
the process is

h h  h h   Ah1h2
2 2

(A) zero (B)  A  1 2  g (C)  A  1 2  g (D) g


 2   2  2
11. The mass m slides down the track and completes the vertical circle on the smooth curved surface. The
minimum value of h will be

R
h

(A) R (B) 2R (C) 2.5 R (D) 3 R

12. Two bars of masses m1 and m2 connected by a non-deformed light spring rest on a horizontal plane.
The coefficient of friction between the bars and the surface is equal to . If P is the minimum constant
force that has to be applied in the horizontal direction to the bar of mass m1 in order to shift the other
bar. Then


(A) P   g m1 
m2
2  (B) P   g m2  m1
2 
g
(C) P   g  m1  m2  (D) P   m1  m2 
2
13. A body of mass m was slowly hauled up the rough hill by a force F which at each point was directed
along tangent to the hill. Work done by the force

Raman Niwas, Near Aakashvani, Mahmoorganj, Varanasi. Ph.(0542)–2363455,website www.catjee.in


[76]
(A) Is independent of shape of trajectory.
(B) Depends upon vertical component of displacement but independent of horizontal component.
(C) Depends upon both the components of displacement
(D) Does not depend upon coefficient of friction.
14. A chain of mass m and length  rests on a rough surfaced table so that one of its ends hangs over the
edge. The chain starts sliding off the table all by itself provided the overhanging part equals     
part of the chain length. If the total work performed by the friction forces acting on the chain by the
moment it slides completely off the table is W and the friction coefficient between table and chain is .
Then
   (1   )2 mg  (1   )mg 
(A)   (B)   (C) W  (D) W 
1 1  2 2
15. A smooth track in form of a quarter circle of radius 6m lies in the vertical plane. A particle moves from
   
P1 to P2 under the action of forces F1 , F2 and F3 . Force F1 is always towards P2 and is always 20 N in
 
magnitude. Force F2 always acts horizontally and is always 30 N in magnitude. Force F3 always acts
tangentially to the track and is of magnitude 15 N. Select the correct magnitude:
6m
O P2

6m F1 
F3

F2
P1
 
(A) Work done by F1 is 120 J (B) Work done by F2 is 180 J
 
(C) Work done by F3 is 45 J (D) F1 is conservative in nature

Raman Niwas, Near Aakashvani, Mahmoorganj, Varanasi. Ph.(0542)–2363455,website www.catjee.in


[77]
Answer Key

Exercise - 1 : [Type – 1]
1. [C] 10. [C] 19. [C] 28. [D]
2. [D] 11. [D] 20. [B] 29. [C]
3. [B] 12. [A] 21. [D] 30. [D]
4. [C] 13. [D] 22. [B] 31. [D]
5. [C] 14. [D] 23. [D] 32. [C]
6. [A] 15. [C] 24. [C] 33. [B]
7. [C] 16. [B] 25. [A] 34. [A]
8. [C] 17. [D] 26. [B] 35. [C]
9. [C] 18. [A] 27. [C]

Exercise - 1 : [Type  2]
1. [A, B] 10. [A,D] 19. [C,D] 28. [B]
2. [A, B] 11. [A,C,D] 20. [B] 29. [C]
3. [A, C] 12. [A,D] 21. [C] 30. [C]
4. [A, D] 13. [B] 22. [C] 31. [A]
5. [A, B] 14. [A] 23. [C] 32. [A,C]
6. [B, D] 15. [B,C,D] 24. [A] 33. [B,C]
7. [B, D] 16. [A,C] 25. [A] 34. [B,C,D]
8. [A,B,D] 17. [A,D] 26. [C]
9. [A, D] 18. [A] 27. [B]

Exercise – 2 [JEE Main/AIEEE Asked]


1. [A] 10. [A] 19. [C] 28. [A]
2. [A] 11. [B] 20. [A] 29. [D]
3. [C] 12. [B] 21. [A] 30. [B]
4. [B] 13. [D] 22. [D] 31. [C]
5. [C] 14. [D] 23. [D] 32. [A]
6. [D] 15. [A] 24. [C] 33. [B]
7. [B] 16. [D] 25. [A] 34. [C]
8. [B] 17. [D] 26. [C] 35. [C]
9. [C] 18. [B] 27. [A] 36. [A]

Exercise -2 [IIT-JEE Advance Asked]


1. [C] 9. [A] 16. [5] 21. [A]
2. 0.098 J 10. [D] 17. [5] 22. [B]
3. [C] 10. [A] 18. [B] 23. [B]
4. [D] 11. [A] 19. A  p,q,r,t; 24. [C]
5. V  3.29 m/s 12. [B] B  q, s; 25. [C]
6. [C] 13. [A] C  p,q,r,s; 26. [A]
7. [C] 14. [D] D  p,r,t
8. [C] 15. [B] 20. [C]

Exercise3
1. [B] 4. [C] 7. [C] 10. [C] 13. [A,C]
2. [C] 5. [A] 8. [B] 11. [C] 14. [B,C]
3. [C] 6. [A] 9. [B] 12. [A] 15. [B,C]

Raman Niwas, Near Aakashvani, Mahmoorganj, Varanasi. Ph.(0542)–2363455,website www.catjee.in

You might also like